Clinical Level Final Mock Exam II

अब Quizwiz के साथ अपने होमवर्क और परीक्षाओं को एस करें!

(Correct) The answer to question 87 A: Rationale:

A, B, and D are all possible answer options. The INITIAL (first) action the social worker should take is (A), report this incident to HR. While generally the first step we take when we have an issue with a colleague is to address this directly with them (C), this is a different situation than addressing a behavior of concern with a colleague. First and foremost, a sexual advance was made and the supervisee may not feel comfortable addressing this directly with the person who harassed them. When sexual harassment occurs, the person who was harassed does not have a responsibility to confront their harasser, even if it is another social worker. Reporting this to HR is a good first step. Second, this isn't an issue of addressing a colleague, but is a supervisor-supervisee relationship where there is an inherent power differential.After doing (A), both (B) and (D) may occur.

(Correct) The answer to question 23 is C: Rationale:

A, B, and D are incorrect because sharing any client information would be a breach of confidentiality. C is correct because the social worker should not share any information with the mother.

Two social workers have been facilitating a 10-week group for men with depression. During the fourth week, the supervising social worker attends the group and becomes concerned regarding the topic of discussion. The supervisor feels that the topic is irrelevant and a distraction for the group. The social worker supervisor should:

A. Address the concerns with the group leaders after the group. B. Encourage group members to share their feelings regarding the discussion. C. Redirect the group to a more beneficial and appropriate topic. D. Address the concerns with the group members directly.

A social worker meets with a new client for an initial session and discovers that the client is going through a divorce with another client whom the social worker has been seeing for 6 months. Both clients are unaware that the other is seeing the social worker for therapy. After the initial session the social worker reviews the Code of Ethics but is still uncertain if it is ethical to provide therapy to both clients. The social worker should NEXT:

A. Address the conflict with both clients. B. Provide the new client with a referral since a relationship is already established with the other client. C. Terminate with both clients. D. Schedule a consultation with a supervisor to address the ethical conflict.

A hospital social worker is assessing a woman who was recently hospitalized due to self-neglect. In developing after care plans, the woman reports that she never married and has no family in the area. In helping the patient develop discharge plans, the social worker should FIRST:

A. Assess the client's ability level to provide self-care at home. B. Refer the client to a local assisted living home. C. Encourage the client to find social support in the area. D. Refer the client to local home health care agencies.

A young boy is raised in a home that presents a stereotyped schema about another social group. When the young man moves away to college, he suddenly finds himself surrounded by people from this group. Through experience and real interactions with members of this group, he realizes that his existing knowledge is completely wrong. This process is an example of:

A. Assimilation B. Accommodation C. Racism D. Stereotypes

A 53-year-old female, married for 25 years, self-refers to therapy as she has been losing weight and not sleeping. She states that her husband screams and yells when he gets very angry and has hit her in the past. How should the social worker initially respond to the client's disclosure?

A. Assist the client in locating appropriate shelters and contact the police. B. Refer the client to a doctor for a medical evaluation and couple's therapy. C. Assist the client in developing a safety plan and refer to psychiatrist. D. Refer the client to a doctor for a medical evaluation and assist the client in developing a safety plan.

A social worker in a community clinic meets with a recently retired 63-year-old man and his wife. During the first meeting, the wife reports that she wants her husband to get help because she is concerned about him, due to recent behavior changes, increased irritability, and difficulty focusing. The FIRST step the social worker should take is to:

A. Assist the client in structuring his time better. B. Explore the client's feelings regarding retirement. C. Provide the client with a referral for a comprehensive physical exam. D. Assure the client it is normal to experience these symptoms during life transitions such as retirement.

A 24-year-old seeks therapy on the advice of his new girlfriend. He reports that he is taking neuroleptics prescribed by his psychiatrist. Neuroleptics are most commonly prescribed for which of the following?

A. Bipolar B. Psychosis C. Depression D. Memory loss

A new middle-aged African American client reports to a social worker that it's been three months since her friend died, but she doesn't feel better and now and it feels like it is getting worse. What is the social worker's BEST action?

A. Inform client that bereavement takes two years to get better. B. Explain that bereavement is an ongoing process. C. Tell client that depression can develop during the process of bereavement. D. Refer client for medication evaluation.

A psychiatric hospital social worker is working with a man who was committed to the hospital after physically assaulting his wife. The man tells the social worker that he blames his wife for how he feels and says, "I would hurt her again if she was here right now." At the end of the conversation, the man says he didn't mean what he said and asks the social worker to not tell anyone. The social worker should FIRST:

A. Inform the treatment team after documenting the threat. B. Call the police to notify them of the threat. C. Address the threat with the client and create a treatment plan. D. Speak with the client's wife to address the threat.

A social worker receives a call from a 34-year-old woman who has been in treatment for two months. She has just been raped by her ex-boyfriend who broke into her apartment while she was sleeping. He left her alone in her apartment and threatened to come back. What should the social worker's initial focus with her be?

A. Instruct her to file a police report and go to the emergency room. B. Help her create a safety plan and file a police report on her behalf. C. Encourage her to file a police report and help her assume responsibility for the rape. D. Suggest that she goes to the emergency room and file a police report on her behalf.

A 54 year old Ukrainian male comes in for treatment complaining of irritability and low levels of energy. He lost his job 2 months ago and has found himself drinking more and more often. He states that he cannot afford paying for therapy out of pocket, however, he can use his wife's insurance that covers him up to 12 sessions. When the social worker looks into the details of coverage, he finds out that the insurance company does not cover treatment for some diagnoses, such as Adjustment Disorder or Substance Abuse, which is precisely the diagnosis given to the client. The social worker should:

A. Keep the diagnosis as is and discuss bartering with the client. B. Change the diagnosis to Depression NOS, which is covered by the insurance company and submit billing for reimbursement. C. Keep the diagnosis as is and discuss other payment options with the client. D. Discuss the situation with the client and allow him to make an informed decision as to what diagnosis to submit to the insurance company.

A social worker in private practice has worked with children for several years when suddenly she loses a child of her own in a car accident. Ever since the social worker has returned from a leave she finds herself drifting off into her own thoughts and memories when working with kids. The social worker notices that she is constantly staring past the children and daydreaming about her child. Which of the following actions should the social worker take?

A. Refer children out of her practice to other clinicians. B. Consult with colleagues regarding countertransference. C. Address grief and loss issues in personal psychotherapy. D. Journal personal process after every session.

A social worker, who is active duty military, is assigned to be deployed. The social worker has not seen one of his clients in over a month and has been unable to reach the client to discuss the imminent deployment. What should the social worker do?

A. Leave a phone message for the client at their home B. Terminate with the client C. Send a letter by mail informing the client and provide referrals D. Reach out to their emergency contact to communicate this information

A 13-year old boy discloses a history of sexual abuse. He informs the social worker that his former social worker touched him during several sessions to demonstrate what is appropriate versus sexually inappropriate touch. What is the social worker's BEST response?

A. Maintain confidentiality B. Confront the social worker C. Report child abuse to appropriate authorities D. Report misconduct of social worker to state licensing board

A social worker has been meeting with a client on a weekly basis for a few months. During the most recent session, the client tells the social worker that he sexually assaulted a woman while walking home last night. In probing the client for further information, the client reports that he has no remorse and is going to do it again tonight. The social worker should:

A. Maintain confidentiality. B. Report the intent based on jurisdictional law. C. Begin to meet with the client twice a week to ensure his violent actions are not acted on. D. Request the client verbally state that he will not go through with his acts.

What action should a social worker take if an adult client reports having stolen a radio from a store?

A. Maintain the client's confidentiality. B. Notify the store manager of the incident. C. Request that the client's family monitor the client. D. Report the client to a local law enforcement agency.

A 24-year-old Asian-American male client seeks therapy for anxiety. In the first session with the social worker he complains of obsessive thoughts that are interfering with his performance in graduate school. The client asks the social worker to teach him techniques that will allow him to complete his assignments. What should the social worker do FIRST?

A. Make a referral for a medication evaluation B. Explore the subject matter of the client's thoughts C. Provide the client with distress tolerance skills to manage the thoughts D. Explore the client's unconscious resistance to completing assignments

A social worker in an outpatient clinic is assigned a Korean American client. The social worker has never worked with the Asian population before and discusses the situation with her supervisor. The supervisor encourages the social worker to be sensitive to which emotional process?

A. Shame B. Rage C. Depression D. Anxiety

A social worker has been treating a Native American woman for 3 months. The social worker has noticed that the woman often tells stories in response to questions asked by the social worker. The social worker should respond to this dynamic by:

A. Refer to a therapist from the client's cultural group. B. Accept the client's form of communication. C. Confront the client about her avoidance. D. Remind the client about her reasons for seeking treatment.

A social worker is in the process of creating a multidisciplinary team for a new mental health clinic. In order to facilitate collaboration between team members, the social worker should:

A. Reinforce the importance of including social work as a component. B. Identify the population of clients that the team will serve. C. Define each member's contribution to the overall team. D. Identify one member to act as the team leader.

A 32-year-old woman with a history of Bipolar II Disorder has been stable on medication for over a year. She arrives at her session with her social worker and is agitated, disheveled, and is speaking rapidly. She reports that she had a terrible cold over the weekend, couldn't sleep, and took an over-the-counter decongestant. The social worker understands her current presentation as:

A. The probable result of her discontinuing her psychiatric medication B. A hypomanic episode induced by the over-the-counter medication C. A typical response to the over-the-counter medication D. A probable progression of her Bipolar II to Bipolar I disorder

A foster care social worker is preparing an update for the court regarding the potential termination of parental rights for the mother of a 6-year-old girl. The social worker has been gathering information from collateral contacts. All of the following would be considered collateral contacts EXCEPT:

A. The therapist running the parenting classes in which the mother is enrolled. B. The child's individual therapist. C. The child's older sister. D. The mother's individual therapist.

A school social worker is hired mid-year after several student suicides at the high school. The social worker discovers that the school's Vice Principal has been the point person for evaluating students who express suicidal ideation, despite not having a mental health background or training. The social worker implements a protocol in which only trained mental health professionals formally evaluate students who express suicidal ideation. All of the following are reasons for this change of protocol EXCEPT?

A. To create a systematized process for responding to students reporting thoughts of suicide B. To prevent future suicides from occurring C. To ensure staff are operating within their scope of competency D. To empower all staff to evaluate students for suicide risk

(Correct) The answer to question 142 is B: Rationale:

Before any of the other options can be done the social worker must FIRST encourage the client to inform his sexual partners of his current diagnosis. If the client refuses to inform his partners it would be important for the social worker to look into the jurisdictional law to assess whether the social worker has a duty to warn an identifiable future partner.

(Correct) The answer to question 126 is A: Rationale:

Before any personal or identifying information can be given, the social worker must consult with the client to determine what information he is comfortable releasing to the group leader. It would be a breach of confidentiality for the social worker to not speak with the client FIRST, therefore A is correct.

(Correct) The answer to question 93 is A: Rationale:

Before anything else can be done the social worker must FIRST evaluate the client's ability to take care of herself at home (A). (B), (C), and (D) may be done at a later time, but the social worker can not develop any other plans before assessing the client's self-sufficiency.

(Wrong) The answer to question 100 is C: Rationale:

Before doing anything else the social worker should discuss her concerns with the physician (C). This allows the social worker to voice her concerns and have a better understanding of why the physician does not want to inform the client of his illness. A may be appropriate only after the social worker has discussed her concerns with the physician. B would be inappropriate at this time. D is also incorrect because it is out of the social worker's scope of practice to inform the patient of the illness before the physician does.

(Correct) The answer to question 19 is C: Rationale:

Collateral contacts are defined as individuals who have knowledge of a family's personal situation without having personal involvement themselves. C is correct because the child's older sister is a family member and has personal involvement in the family and would NOT be a collateral contact.

(Wrong) The answer to question 123 is C: Rationale:

Complaints and concerns should be discussed with a direct supervisor who is able to listen to the concerns and address them as needed (C).

(Correct) The answer to question 80 is B: Rationale:

Countertransference is defined as a set of unconscious or conscious emotional reactions to a client experienced by the therapist. In this case it is seen through the social worker's cravings and anger towards the client. Therefore, B is correct because the social worker needs to immediately address the countertransference issues by consulting with a supervisor. A is not necessary yet unless the social worker is unable to resolve the countertransference issues. The social worker is only experiencing cravings and has not actually started to use again. While it may be helpful to obtain personal assistance with maintaining sobriety, it is not necessary to return to alcohol treatment at this time (C). D would be inappropriate for the social worker to do.

(Correct) The answer to question 111 is B: Rationale:

During the beginning phase of treatment it is most important for the social worker to clarify the expectations of the client and what they want out of treatment (B). A, C, and D, may all be done at a later point.

(Correct) The answer to question 165 is D: Rationale:

Empowering the clients to meet with local officials regarding their needs allows for them to advocate for their own rights and enables their sense of self-determination (D).

(Correct) The answer to question 69 is C: Rationale:

Ethically, social workers seek appropriate professional assistance for their personal problems or conflicts that impair work performance (C). Consulting with colleagues (B) or journaling (D) do not address the severity of the situation. If the social worker is still unable to work after seeking professional assistance then she would have to refer her clients out (A).

(Correct) The answer to question 67 is C: Rationale:

(C) is the MOST appropriate solution as it allows the inventory to continue to be used while actively engaging clients in the process and removing the time outside of sessions the social workers were using to score these measures. Concerns about the measure itself have not been reported, making (A) not our best option. (B) may not be possible. (D) may not be feasible and does not offer a long term solution to the issue.

Which of the following type of progress notes looks at the subjective report of the client, the objective observations of the therapist, an assessment of the client's diagnosis/presenting condition, and a plan to address the presenting issues?

A. Progress notes B. SOAP notes C. Process notes D. DAP note

(Correct) The answer to question 97 is A: Rationale:

A PRIMARY prevention effort involves a social worker creating services that target a problem and prevent the onset of that problem. A is not a primary prevention strategy because the children have already been diagnosed with ADHD and therefore the group is not preventing the disorder, but rather helping these kids who already have ADHD develop social skills.

(Wrong) The answer to question 3 is B: Rationale:

A client will generally most likely follow through with a referral if he is aware of how that agency or referral will best help (B). A and D are only trying to persuade the client to use the referral in a manner that doesn't allow the option of the client deciding what is best for him; self-determination is always key! C gives the client the option to not use the referral which is not what the social worker is trying to encourage the client to do.

(Wrong) The answer to question 131 is B: Rationale:

A genogram (B) is used to depict family relationships over at least three generations. In this case it would allow the social worker to understand the relationship between the couple's current problems and their family history. A, C, and D are incorrect because none of those options help the client's understand the impact that their parents have on their present situation.

(Correct) The answer to question 145 is A: Rationale:

A is correct because client confidentiality is always maintained unless there is the potential for harm to self/others, which in this case there is not. B, C, and D would all be a breach of confidentiality.

(Correct) The answer to question 154 is A: Rationale:

A is correct because it is the most comprehensive assessment of a client who is from a different culture than the social worker. Socio-economic variables would assess variables such as income, education, and occupation. Determining culturally-related and acculturation issues is important in order to have a better understanding of any difficulties the client is experiencing on a cultural level from the client's perspective. B is incorrect because it is making assumptions regarding the client's beliefs (religious and the use of traditional healers) that may not be true. C and D are incorrect because evaluating the client's mental status can have problems when the client and social worker are from different cultures. The social worker may make assumptions regarding the client's affect, behavior, and emotions that are actually culturally normative for the client.

(Wrong) The answer to question 15 is A: Rationale:

A is correct because the social worker must ethically FIRST document the threat and then notify the treatment team of what the client said during the session. B, C, and D may all be done if deemed necessary once A has been completed.

(Correct) The answer to question 66 is A: Rationale:

A is correct because when working on a multidisciplinary team, client's should always be made aware that confidential disclosures are disclosed within the team in order to provide continuity of care.

(Correct) The answer to question 117 is C: Rationale:

A is incorrect because it would be unethical to ignore the problem. B is incorrect because the issue has not been properly addressed with a supervisor first before a report is made to the licensing board. D is incorrect because the situation should be addressed with the colleague and a supervisor rather than with the client directly. C is correct because the social worker needs to speak with a supervisor about the situation so that it can be properly handled in an ethical manner.

(Correct) The answer to question 14 is A: Rationale:

A is the best answer because it helps the social worker assess potential risk. B might be appropriate if the client's clarification indicated the necessity to warn the authorities and the partner. C is not the best answer because the client's statement does not reach the level of severity or specificity needed for Tarasoff. D might be an appropriate intervention but not until after the therapist has identified the level of risk.

(Wrong) The answer to question 58 is A: Rationale:

A is the best answer due to the symptoms of depression the client presents with assessing for suicidality is first. B, C, and D are all things the social worker would do in this scenario but not first.

(Correct) The answer to question 169 is D: Rationale:

A medical exam should always be performed FIRST to rule out any medical problems that may be related to changes in the client's health. While the other options may need to be done at a later time, the social worker needs to first have the client seen by a physician who can address the medical concerns.

(Correct) The answer to question 102 is C: Rationale:

A social worker should always address the issues FIRST (C) with the colleague before taking the situation to a higher authority. If the social worker voices her concerns and the situation continues to proceed it would then be appropriate for the social worker to take further actions and be in contact with a supervisor.

(Correct) The answer to question 91 is B: Rationale:

A solutions-focused intervention would be MOST appropriate to use here because it allows each sibling to have separate goals based on their own needs (B). A may be important to do once the service plan has been created, but the foster parents' concerns are not the MOST important part of creating a service plan. C would disregard the individual needs of each child. D is unnecessary to do since some of the children's needs and goals may not be related to why they are in foster care.

A client is ready to terminate therapy. Which of the following questions would the social worker answer to determine the client's readiness to terminate?

A. "Have the troublesome behaviors in the relationship been stabilized?" B. "Have all the referrals been attempted, discussed, and eliminated?" C. "Is the client able to use what has been learned in treatment?" D. "Is the client willing to interpret self-generated behaviors?"

During a therapy session, a client states that he recently found out his partner is having an extramarital affair. He says he is furious and so angry that he wants to do something to "get back" at his partner. What should the social worker do FIRST?

A. Ask questions to clarify the client's statements. B. Remind the client about the limits of confidentiality. C. Notify the authorities and the partner according to the Tarasoff law. D. Assess the client for history of violent behavior.

A social worker was recently promoted to Director of Prevention Programs at a community mental health agency. The agency was awarded a grant to develop primary prevention strategies for the families they serve. The social worker should focus on all of the following EXCEPT:

A. A social skills group for children with ADHD. B. Free informational sessions on immunizations for parents. C. Ongoing skills training for teens on how to build healthy relationships. D. Providing information on infant development to new parents.

The parents of a 4-year-old boy bring him in to meet with a social worker due to ongoing behavioral difficulties. The mother reports that they adopted their son when he was 13-months-old from an orphanage in Eastern Europe where he was severely neglected. The mother begins to cry, reporting that since then their son won't let them hug him and states that he, "often prefers the comfort of strangers over us." The father says that the boy doesn't engage with other kids his age and often appears withdrawn. The MOST likely diagnosis for this child is:

A. ADHD B. Major Depression C. Reactive Attachment Disorder D. Oppositional Defiant Disorder

A social worker in private practice works predominantly with adolescents and uses text messaging to remind them of upcoming appointments. The social worker accidentally sends a message to a client that was intended for her son that says, "I love you." What is the BEST response to this incident?

A. Acknowledge mistake and discuss the boundary violation B. Explore the impact of texting on the therapeutic relationship C. Stop using text messaging in clinical practice D. Send a follow-up text apologizing for the mix up

A new client comes for therapy complaining of anxiety and depression. After the social worker provides information about the benefits of medication and psychotherapy, the client emphatically states, "I refuse to take medication." What should the social worker do NEXT?

A. Acknowledge the client's right to choose his treatment. B. Refer the client to a psychiatrist. C. Determine why the client is so against medication. D. Provide the client with research on the benefits of medication.

A 13-old-boy is referred to therapy by his middle school counselor because he is disruptive in class and displays signs of oppositional defiance. His middle-aged mother attends the first session wearing a Muslim headscarf. The social worker is wearing a Jewish Star of David necklace. The mother notices the necklace and inquires whether the social worker is Jewish. What is the social worker's BEST response?

A. Acknowledge the mother's question and remind her that they are there for her son B. Acknowledge the mother's question and provide an honest answer C. Acknowledge the mother's question and explore its significance D. Acknowledge the mother's question and remove the necklace

After meeting with a Japanese-American family for several months, the family has experienced improvements in communication and their children's' behavior has improved. The parents give the social worker a gift card in appreciation of the good work with the family. What should the social worker do NEXT?

A. Acknowledge the parent's appreciation and accept gift B. Acknowledge the parent's appreciation and decline gift C. Determine whether "fee for service" is acceptable in area D. Explore cultural importance of providing gift

A hospice social worker is meeting with two women whose father is dying. The social worker should FIRST:

A. Address end-of-life wishes. B. Provide referrals to local agencies that can assist with funeral arrangements and costs. C. Encourage the daughters to express unresolved issues to their father. D. Help the daughters understand what to expect during the grief process.

A social worker has recently discovered that a colleague at the same agency has developed a platonic outside relationship with a client's spouse while providing couples therapy for them. The social worker's FIRST action should be to:

A. Address the ethical violation with the client's spouse. B. Meet with a supervisor to address the ethical violation. C. Meet with the colleague to address the potential ethical violation. D. File a complaint with the licensing board.

A social worker has been treating a client for 6 months when the client states that she will be leaving town for a few months. The client asks if she can use email to keep in touch with the social worker while she is away. The social worker should FIRST:

A. Address the limits and risks of using email with the client. B. Provide the client with a referral to a therapist in the area where the client will be living. C. Discuss the option of doing weekly phone sessions while the client is away. D. Discontinue therapy until the client is back.

A social worker at a community mental health clinic becomes aware that the clinic plans to participate in a research study around early-onset Schizophrenia in which some prospective clients are assigned to a treatment protocol while others are put on a wait-list until the trial concludes. What should the social worker do FIRST:

A. Address the unethical nature of the study with her supervisor B. Begin contacting prospective clients as instructed C. Refer clients on the wait-list to other agencies who can provide treatment D. Contact the state board about the ethical issue

A 20-year-old woman has been seeing a social worker due to restrictive eating behaviors, significant weight loss, and compulsive exercise. The client has been unable to gain weight and lost her menstrual cycle six months ago. The client reports that these symptoms started after she was sexually assaulted near her apartment. She says that she is afraid to go out at night now, and finds it difficult to sleep because she "sees it happening over and over again in my head." The MOST likely diagnosis is anorexia nervosa and:

A. Adjustment disorder B. Nightmare disorder C. Posttraumatic stress disorder D. Amnesic disorder

A social worker is developing a new program for high school aged kids with Autism Spectrum Disorder. After researching effective therapy models for this population, the social worker has determined that a social skills learning group would be the best approach. Which of the following steps should the social worker take NEXT?

A. Appoint group leaders who have experience working with this population. B. Create a weekly curriculum for the group. C. Develop a plan that measures the effectiveness of the group in teaching social skills. D. Post flyers in local high schools to recruit interested kids.

A hospice social worker obtained a referral for a 78-year-old man whose health is quickly deteriorating. In speaking with the patient over the phone, the man states that he only wants to receive hospice services if he can be discharged to his son's home. However, in speaking with the son, he reports feeling like he would not be able to care for his father properly. The social worker should FIRST:

A. Arrange to meet with the family to address the patient's end-of-life wishes. B. Speak with the son to address his hesitations. C. Schedule a consultation with a supervisor to address the conflict. D. Meet with the patient to encourage inpatient care.

A social worker is leading a life skills class for individuals with physical disabilities. During the class, the clients state their disappointment due to the lack of handicapped accessible train stations in the city they live in. The social worker should:

A. Assist the clients in finding alternative forms of transportation. B. Encourage the clients to meet with a lawyer to address their concerns. C. Encourage the clients to use the buses rather than the trains. D. Encourage self-determination by empowering the clients to meet with local officials to address public transportation accessibility.

A social worker at a community agency has been meeting with a 15-year-old boy for one month. During the most recent session, the boy discloses that he still meets up occasionally with his previous social worker. The social worker should should:

A. At the next session, inform the boy that the previous relationship must be terminated. B. Have the client sign a release allowing the social worker to contact the previous social worker. C. Terminate therapeutic relationship. D. Speak directly to the boy about the conflict.

A social worker at a residential treatment center has been providing case management to a client. While working with other clients the social worker has heard that the client is dissatisfied with the case management services the social worker is providing and said the social worker doesn't care about her. The social worker should:

A. Be more self aware of how the social worker presents herself in front of this particular client. B. Meet with the client to address what was said and encourage her to be more direct about her concerns regarding the social worker in the future. C. Transfer the client to a different social worker in the agency. D. Meet with the client individually to address the rumors that were heard and facilitate a non-confrontational discussion regarding the problem.

A hospital social worker is working with a 45-year-old man who was recently hospitalized. The social worker is part of a multidisciplinary team at the hospital and in speaking with the patient's physician learns that the patient has a terminal illness. The physician states, "I don't think it is appropriate to inform the patient of his prognosis. It will only make his final days more difficult." The social worker, who disagrees with the physician, should:

A. Call for a meeting with the treatment team. B. Contact the medical licensing board to report the physician's unethical practices. C. Address the concerns with the physician directly. D. Meet with the patient's family first to discuss the prognosis before telling the patient.

A school social worker is referred a 16-year-old girl for missing class. She has been skipping classes, is often moody and is failing to turn in assignments. During the session, she tells the social worker that she is absent from school because her mom drinks a lot and will forget to pick up her sister from kindergarten. The social worker should FIRST:

A. Call the mother and schedule a session. B. Identify if there are other people to help with childcare. C. Refer mother to substance abuse treatment. D. Call Children's Protective Services.

A hospital social worker is preparing a discharge plan that includes a referral to a community mental health agency for counseling. The client will MOST likely follow through with the referral if the social worker:

A. Clearly states the consequences of not using the referral. B. Outlines the benefits of the agency and how the agency can help the client. C. Ensures that the client understands that following through is optional. D. Informs the client that a staff member at the agency will be waiting to hear from him.

The parents of a 16-year-old boy make an appointment to meet with a social worker due to ongoing arguments at home. During the initial meeting, the boy reports feeling angry that his parents do not listen to him or value his feelings. His mother agrees, saying that she wishes they were all able to communicate better with one another. In creating a treatment plan, the social worker should write the objective as:

A. Communication skills will improve for the majority of the week. B. No more than two arguments will be reported per week. C. Communication skills will improve 75 percent of the time. D. No arguments will be reported by the client during the week.

While assessing a 71-year-old man recently admitted to the hospital following a stroke, the medical social worker notes that the client is confused and unable to answer simple questions. The wife states that he isn't thinking clearly. The social worker should NEXT:

A. Complete a mental status exam. B. Collect a psychosocial history from the wife. C. Notify the medical team that he is not competent to make decisions. D. Obtain a release from the patient to speak to the wife.

A social worker sees a 75-year old Asian American man whose wife died six months ago. In the intake interview, the client reports that he has lost weight, has very little energy, and has difficulty getting himself to do things. What should the social worker do FIRST:

A. Complete a suicide risk assessment of the client B. Explore available family and community resources C. Suggest the client schedule an appointment with his doctor D. Refer the client to a local bereavement group

A supervisor using a Human Relations management approach is interested in improving staff productivity. Which of the following actions will the supervisor MOST likely take?

A. Conduct a complete study of the most productive employee and use to create a standard protocol. B. Meet with staff in small groups and explore their current needs and motives at work. C. Implement a program of regular training and education around principles of productivity. D. Provide staff with a document clearly stating expectations regarding productivity and tasks to help staff achieve this goal.

A new client meets with a social worker. The client has a history of multiple suicide attempts and self-harming behaviors. During the assessment, the social worker asks questions about the client's trauma history. The client discusses an extensive history of abuse by his father, though while he is talking, he often smiles or laughs. This behavior is best defined as:

A. Congruence in communication B. Incongruence in communication C. A defense mechanism D. A sign that he isn't ready to deal with his trauma yet

A social worker at a psychiatric hospital has been working with an adult man to prepare him for discharge. In speaking with the client, the social worker learns that the client has a limited family and social network. To help the client successfully discharge and re-enter the community, which social work activity will BEST support the client in this transition?

A. Connecting with agencies in the community B. Connecting the client to financial assistance resources C. Encouraging the client to join a support group D. Assisting the client in applying to local housing

After working with a client for three months, a social worker notices that she is having trouble remaining impartial to the client and not letting her own values interfere with the treatment. The social worker's BEST response is to:

A. Consult with colleagues. B. Discuss the case with a supervisor. C. Address the value conflict with the client. D. Provide the client with referrals to another social worker.

A social worker is contacted by the local police department requesting the records of a client who recently committed suicide. What is the BEST response for the social worker?

A. Consult with their supervisor B. Turn over a copy of the client's record to the police department C. Not share the record under any circumstances D. Contact the client's legal representative to determine how to proceed with the request.

A new student at a high school is told by a school administrator that she cannot wear her ethnic headdress to school. The student is very upset and is sent to the social worker's office. After processing the situation with the student, what should the social worker do NEXT?

A. Contact her parents and inform them of situation. B. Advocate for the client by addressing the issue with the principal. C. Host a diversity training with the administration. D. Suggest that the student transfer to another school.

During a session with a client, a social worker learns that a family member recently disciplined her child with spanking that left red marks. The social worker MUST:

A. Contact the family directly to assess if abuse is occurring or not. B. Disregard the information since the child is not the social worker's client. C. Make a suspected child abuse report. D. Refer the family member to anger management classes.

A social worker meets with a 14-year-old client who enters the session in tears. The client says she is afraid for her 15-year-old best friend, who lives in a group home. The client says that her friend recently had a black eye and marks on her wrists that appear to be from straps. When the client asked her friend about these marks, her friend changed the subject, but the client says that her friend has become more withdrawn lately. The social worker working from a psychodynamic perspective should NEXT:

A. Contact the group home supervisor to address ethical consideration. B. Role play with the client another conversation she should have with her friend about the possibility of abuse. C. Help the client experience a cathartic moment regarding the situation. D. Validate the client's concerns and make a child abuse report.

A couple present with a history of conflict. They accuse each other of behaving like their respective parents. Which of the following interventions could the social worker use to help the couple understand the impact of their parents on their present situation?

A. Create an enactment of one of their arguments. B. Construct a genogram of at least three of their generations. C. Reframe their conflict as each attempting to control the other. D. Interpret their conflict as an expression of abandonment anxiety.

A social worker in a community agency is meeting with a 67-year-old woman who tells the social worker that she has lost interest in her spouse after 40 years of being happily married. The client also reports feeling disconnected from friends and agitated during the day. The woman states that she worked as a nurse for many years but stopped working after a double mastectomy and subsequent chemotherapy, which concluded 2 months ago. The social worker should FIRST:

A. Determine when the client's problems first began. B. Refer the client for psychiatric evaluation. C. Obtain a signed release to consult with client's doctor. D. Refer the client to an exercise program.

A social worker begins working with a 20-year-old client who is experiencing depression. The role of the social worker during this beginning phase of treatment is BEST understood by:

A. Determining the most appropriate therapeutic interventions to use. B. Clarifying and exploring the client's expectations. C. Creating a treatment plan to be followed throughout the course of treatment. D. Addressing the importance of the therapeutic relationship.

A young mother's children were put in foster care after neglect was reported by a neighbor. The child protection social worker has been meeting with the mother and together they developed a plan in order to help her regain custody of her children. The client reports that she is eager for her children to return home, however, she does not follow the plan that was created. The social worker should FIRST:

A. Develop smaller, more manageable tasks with the client. B. Encourage the termination of parental rights. C. Confront the client regarding her lack of progress. D. Meet with the mother to explore her understanding of the tasks in relation to the goal of regaining custody of her children.

A 45-year-old man seeks treatment from a social worker in private practice due to ongoing difficulties at work. During the assessment, the client reports that he has trouble getting along with his co-workers, does not have any "real" friends, and has never been in an intimate relationship. When the social worker attempts to explore his family history, the client becomes very quiet, appears distracted, and requests to talk about something else. From a trauma-informed perspective, the social worker should:

A. Diagnose the client with Schizoid Personality Disorder B. ​​​​​​​Gently assess for further signs of early childhood abuse and/or neglect C. Avoid asking questions about the family until the client is ready to discuss them D. Refer the client to an adult social-skills group

A social worker in private practice is using a very structured treatment model that includes mindfulness activities and instructions to contact the social worker if clients have any thoughts of self harm. What model is the social worker using?

A. Dialectical Behavior Therapy B. Psychodynamic therapy C. Systems therapy D. Interpersonal psychotherapy

A 53-year-old military veteran is experiencing a great deal of frustration and anger in his relationship with his 12-year-old son. He complains, "My son doesn't listen to me. I know my wife is turning him against me." How would a solution-focused social worker intervene to assist this client?

A. Directly realign the executive subsystems with expectations for change. B. Devise strategies to eliminate symptoms, thereby leading to change. C. Collaboratively design a solution with expectations for change. D. Rewrite the problem for the client, thereby leading to change.

While working in the hospital, a social worker is approached by the mother of a former patient. The mother states that she lost her son to cancer and is interested in knitting blankets for children in the hospital who have cancer. The mother asks if the social worker can tell her the names of the children who have cancer so she can personalize the blankets with their names. The BEST response for the social worker is to:

A. Disclose the names of the children. B. Encourage the mother to speak with an oncology nurse. C. Inform the mother that this information cannot be given out. D. Encourage the mother to speak with the families directly.

A social worker is part of a multidisciplinary team at a residential drug and alcohol treatment center. During the initial interview session, clients should be made aware that confidential disclosures are:

A. Disclosed to other members of the treatment team. B. Kept solely between the social worker and the client. C. Disclosed only when the client expresses harm to self or others. D. At no point shared with others.

A 34 year-old woman seeks treatment for increasing symptoms of anxiety. She is the single mother of a 4 year-old girl, after having separated from her daughter's father 3 months ago. She reports that their separation was mutual and they continue to be friends, but she states that her feelings of anxiety are increasing. She is increasingly concerned about her ability to be a single parent and has recently been considering sending her daughter to live with the father. She reports feeling disconnected from her daughter and unsure of how to "get things back to normal." What should the social worker do FIRST?

A. Discuss a referral to a psychiatrist for medication evaluation. B. Offer a joint session with client's ex to discuss ways to split the daughter's time between the two of them. C. Assess whether there have been any recent incidents that contributed to client's increased anxiety. D. Acknowledge client's feelings of distress and confusion.

A 45-year-old lawyer comes to his session drunk. The social worker's INITIAL response would be to:

A. Discuss his decision to drink before the session. B. Determine whether he was driving drunk. C. Reschedule the session and assist him in finding a safe way home. D. Call someone to come pick him up.

A 42-year-old woman has been in therapy for depression for two years. She has met the mutually agreed-upon goals. She requests ongoing therapy because she does not want her life to go back to the way it was. What course of treatment is most indicated at this point?

A. Discuss termination issues and maintain regularly scheduled sessions. B. Discuss termination issues and gradually decrease the frequency of contact. C. Reformulate goals and address anxiety inhibiting client autonomy. D. Reformulate goals and normalize client's fears of independent living.

A social worker has been conducting home visits with a young mother who has a 10-month-old baby. During the most recent visit, the mother reports she is worried that her child is unable to walk yet, explaining that her sister's child started taking her first steps around this age. The social worker observes that the baby is able to pull himself up to a standing position while holding onto a table. The BEST response for the social worker is to:

A. Educate the mother on normal child development. B. Encourage the mother to take her baby to see the pediatrician for an evaluation. C. Encourage the mother to join a group for mothers and babies. D. Increase the number of home visits to monitor the child's development.

A couple brings their adult son to meet with his social worker. The son is diagnosed with schizophrenia and lives with them. The couple reports that the previous night the son assaulted the father and now they are afraid of him. They are not sure he is taking his medication. In the interview, the social worker finds that the client is psychotic, agitated, and unwilling to be hospitalized. What should the social worker do NEXT?

A. Encourage client's father file a police report of the assault. B. Work toward emergency psychiatric commitment. C. Assist the parents in filing a restraining order. D. Attempt to convince the client to take his medication.

A social worker runs an anger management group at a community mental health agency. A group member meets with the social worker individually, to complain that another group member shared confidential information outside of the group. What is the social worker's BEST response?

A. Encourage the client to address concerns during the next group session B. Address the issue with the other group member at next group session C. Remind group members of the importance of maintaining confidentiality D. Meet with the offending group member and discuss concerns

A new client reports that her husband is always yelling at her and saying that he does not know why he married her since there are so many other women who would want to be with him. The client states that she knows he's right and she wants the social worker to help her learn how to be a good wife for him. The social worker should FIRST:

A. Encourage the client to attend marital counseling with her husband. B. Explore ways in which the client can communicate better with her husband. C. Discuss the common dynamics of domestic abuse with the client. D. Provide the client with referrals to domestic abuse shelters.

Due to having limited family in the area, an elderly client lives alone and has been receiving home health services. The social worker is meeting with the client to evaluate him for possible dementia after a nursing aide expressed concerns. The nurse reported that the client has become disoriented, is unsteady while walking, and is having difficulty communicating. What should the social worker do FIRST?

A. Encourage the client to be moved into an assisted living home. B. Increase the amount of care the client is receiving. C. Encourage the client to relocate to an area closer to family who can assist her. D. Have the client be evaluated by a physician.

A client with a diagnosis of major depression meets with a social worker. The client reports that she has been in treatment many times before, but says, "my depression has never gotten any better." While developing the treatment plan, the client agrees with the social worker's suggested objectives saying, "I will do anything to make this go away." Effective treatment planning would suggest that the social worker should:

A. Ensure the client's understanding of the treatment process. B. Help the client identify barriers to success in previous treatment attempts. C. Suggest that the client set her own objectives. D. Discuss evidenced-based treatment protocols for major depressive disorder.

A social worker is leading a support group for parents who lost children due to drug overdose. During the group, the social worker brings up her own experience of losing a child to drugs. This use of self-disclosure allows for the social worker to:

A. Establish herself as an effective group leader. B. Illustrate a sense of understanding. C. Illustrate an understanding of the effects of addictions. D. Receive support from group members.

A pregnant client is referred to therapy after she tested positive for cocaine. The client states that it was a one-time event and she doesn't have a problem. The social worker is utilizing a stage of change model. During the precontemplation phase of treatment, the social worker would do all of the following EXCEPT:

A. Establish rapport and trust. B. Explore events that precipitated treatment. C. Commend the client for entering therapy. D. Explain that the client needs to be open to change for therapy to work.

Which of the following factors should be included in the assessment of a client from a culture that is different from the social workers?

A. Evaluation of socio-economic variables, determination of any culturally-related issues, and determination of level of acculturation. B. Evaluation of socio-economic variables, consultation from traditional healers, assessment of religious beliefs. C. Evaluation of mental status, determination of any culturally-related issues, assessment of religious beliefs. D. Evaluation of mental status, determination of any culturally-related issues, and determination of level of acculturation

A therapist is facilitating an anger management group for men who have been court-ordered for treatment due to domestic violence charges. During the initial session, the group members ask the therapist questions about confidentiality and how much information from group will be shared with the court. The therapist should:

A. Explain that the court will only be provided with general treatment summaries B. Explain that all information from groups will be shared with the court C. Review the requirements from the court to identify what information will be shared D. Encourage clients to only share information they feel comfortable being passed on to the court

A couple brings their 16-year old son to a social worker for a first appointment. The mother is concerned that the son has been withdrawn recently and seems depressed. The father reports that he recently found homoerotic material on his son's computer. The parents consent to the son's treatment and leave, but the father rushes back and demands to know, "When will you be able to tell me if my son is gay?" The son, avoiding eye contact says, "I don't want to be here, you won't understand me." What would be the social worker's BEST response?

A. Explain the limits of confidentiality to the son and parents. B. Explain to the parents that confidentiality prohibits the social worker from sharing information. C. Speak to the son privately and explain to him the limits of confidentiality. D. Validate the son's concerns about being understood.

A social worker has been trying to become more culturally aware due to an increasing amount of diverse clients at work. She has been reading about communication practices, values, and beliefs of various ethnic populations. In order to continue in becoming competent in multicultural practices, the social worker should also:

A. Explore one's own cultural beliefs and prejudices. B. Seek out personal psychotherapy to address explore personal beliefs about other cultures. C. Immerse oneself in the culture that is being studied. D. Develop an understanding of the communication patterns used by diverse cultures.

A young woman comes into therapy upset because she was passed over for a job because she expressed interest in starting a family. What should the social worker do NEXT:

A. Explore the client's feelings about the potential discrimination. B. Refer her to get some vocational training. C. Validate her anger and refer her to a lawyer. D. Suggest she speak to her employer.

A social worker at a community mental health clinic is working with a client whose partner is physically abusive towards her. In speaking with the client, the social worker notices that she often makes excuses for her partner, stating that he is just stressed out about work. The social worker has complied with reporting requirements and despite encouraging the client to go to a shelter, the client refuses. What should be the social worker's NEXT step?

A. Explore the client's past relationships to address abusive patterns. B. Explore positive coping mechanisms the client can use. C. Encourage the client to attend couples therapy with her partner. D. Develop a safety plan with the client.

A social worker at a family mental health clinic sees a young boy whose parents have noticed recent behavioral changes in him. They report that he no longer hangs out with his friends at school, does not enjoy the same activities he used to, and appears withdrawn. Throughout the interview it becomes apparent that the changes occurred after the parents and child moved into a family friend's house. The social worker should NEXT:

A. Explore the recent changes in the home environment. B. Inform the parents that these changes in behavior are developmentally normal. C. Address the possibility of child abuse with the parents. D. Encourage the parents to find a new living arrangement for the child.

A social worker in private practice meets with a 32-year-old man for an intake. The man reports a long history of work issues involving productivity and task completion. He reports having done well in school and says that his relationships are stable. The social worker completes the social history to:

A. Facilitate a referral for job training and support. B. Understand factors that affect the client's behavior. C. Identify client's social support system. D. Assess current coping mechanisms.

A hospital social worker is part of a multidisciplinary treatment team on an orthopedic unit. Part of the social worker's duties involves recording notes in the patients' medical records. The PRIMARY purpose of this documentation is to:

A. Facilitate the appropriate discharge plans when necessary. B. Demonstrate to the hospital that appropriate services are being provided. C. Ensure continuity of care once the client has discharged. D. Allow for appropriate communication to occur between team members regarding the care of patients.

Parents seek treatment for their 21-year-old daughter because they are concerned about her binge eating. What is the most accurate source of information when diagnosing eating disorders?

A. Family history B. Self reports by client C. Eating inventory D. Behavior observation log

An 85-year-old woman has been in the hospital due to ongoing medical problems. The woman lost her husband 5 years ago and has since then been living on her own with the help of family who live nearby. The hospital social worker meets with the physician who states that the client will need to move into an assisted living home. The physician reports that the client will be angry with this plan and therefore asks that the social worker arrange for the move to occur without the client's knowledge. The social worker should tell the physician that:

A. Family members should be given the option to have the client live with them. B. The family should be in charge of finding the appropriate placement. C. The client should be a part of the process in deciding what happens next. D. The client should be given the option to select various discharge plans.

Which of the following factors are key to the initial evaluation of an adult client who has no housing and who presents with fatigue and difficulties in sleeping and eating?

A. Family relationships and support systems. B. Health status and ability to provide basic care. C. Social isolation and interpersonal functioning. D. Financial status and ability to access resources.

A 9-year-old boy is brought to therapy by his parents. They were referred by his school because he is bullying other kids. The father states angrily, "I'm tired of dealing with his crap. His mom is too nice to him. I'm the only one who's willing to lay down the law." The mother appears frightened and does not make eye contact with you. The boy hits her on the arm. The father grabs him and says, "I'm the only one who can do that." What should the social worker do FIRST?

A. File a child abuse report. B. Ask Mom about her parenting style. C. Interview each family member separately. D. Determine when the problematic behavior started at school.

A client seeks therapy after the death of her mother. The client shares that her 82 year-old mother was living with her when she died. The client shares that her mother had dementia and would frequently become verbally and physically aggressive to her and her children. She tears up when she reports that she would be so desperate for a break that she'd sometimes lock her mom in her bedroom for an entire day. The social worker should NEXT:

A. File an abuse report with Adult Protective Services B. Assess for child abuse C. Acknowledge her feelings of shame D. Refer her to a grief support group

A community agency uses the Beck Depression Inventory to monitor client progress on a weekly basis. The social work team lead is concerned about the amount of time it is taking for staff to score and input this measure. After meeting with her team, the social work team lead determines the time it is taking to score the inventories makes it difficult to complete within their scheduled work week. What is the MOST appropriate solution for this problem?

A. Find a measure that takes less time to score B. Hire additional social workers C. Encourage the social worker and client to collaboratively score and review the inventory in session D. Offer overtime to ensure social workers score and input data.

A social work supervisor has their supervisee keep a written record of an interaction with a client that does not go into the client's official record. The social worker and supervisor use this to examine the dynamics of a particular interaction for the purposes of learning and refining skills can best be defined as a:

A. Progress notes B. Process Recording C. Clinical Supervision D. Clinical Documentation

A new client comes to session displaying lethargy and describes her situation as "near hopeless." She failed her real estate examination two times. She tells the social worker that no matter what she does, there is no way she can pass that examination. She also says that she's been gaining weight since she's been studying. If the social worker was using Cognitive therapy, which of the following would constitute elements of the treatment plan?

A. Forming a collaborative relationship; working on her misperceptions of physical symptoms; using Socratic questioning. B. Forming a collaborative relationship; working with her negative view of the cognitive triad; working on her underlying assumptions that are fueling her automatic thoughts. C. Forming a strong therapeutic relationship; working on her fear of being fat; employing collaborative empiricism. D. Assuming a directive stance; working on themes of loss and defeat; exploring her underlying negative schema.

A couple in their 40's, are self-referred and want to work on their marriage. The wife reports that they have been married 25 years and that things are just not the same anymore. The husband says that they hardly talk to each other and he misses the good relationship they once had, what interventions would a social worker consider if using Strategic Family Therapy?

A. Give directives, explore the role that past family relationships are playing in the present, model and educate about communication skills. B. Do a genogram, utilize restraining and positioning strategies, comment on patterns of each person attempting to control the relationship. C. Give directives, utilize restraining and positioning strategies, model and educate about communication skills. D. Do a genogram, explore the impact of past family relationships on the present, model and educate about communication skills.

A school social worker starts a social skills group for middle-school girls that is aimed at increasing positive peer interactions and improving self-esteem. During the first session, the social worker provides the students with information about the group and has each member say her name and grade. To create group cohesion, the social worker should NEXT:

A. Have each group member describe a recent conflict with a peer B. Meet with each student individually to build rapport with the social worker C. Facilitate a group discussion about rules and expectations D. Share a personal experience related to low self-esteem

An 8-year-old boy is constantly falling asleep at his desk during school and often arrives to school in dirty clothes. Due to these concerns, the boy's teacher refers him to the school social worker. The social worker should FIRST:

A. Have the school nurse evaluate the boy. B. Make a report to Child Protective Services (CPS). C. Refer the child to a community mental health center that specializes in children. D. Set up a meeting with the parents and the child.

A family meets with a social worker for family therapy due to concerns regarding their 16-year-old daughter. The mother reports that their daughter was recently diagnosed with a medical condition that requires medication monitoring and a strict diet. She says that her daughter has many friends, and does well in school, however has recently been questioning her parent's authority and demanding to make her own choices. The father also reports that their daughter refuses to follow her required diet. This behavior is MOST likely due to?

A. Her parents misunderstanding of typical adolescent behavior. B. The client is experimenting with drugs and alcohol. C. The client's need to exert control over a part of her life. D. Noncompliance with medication in combination with irregular diet.

A social worker at a day treatment program notices a colleague giving the agency credit card to a client so that he can go next door and purchase supplies for the office. When the social worker brings up the situation with the colleague, the colleague states that the client always brings back a receipt and has never used the card inappropriately. The social worker should:

A. Ignore the problem since the colleague was already addressed. B. Report the colleague to the licensing board. C. Notify a supervisor of the situation so it can be further addressed. D. Inform the client that he should no longer be doing favors for the colleague.

A 62-year-old client who is 3 weeks sober from alcohol reports difficulty performing sexually with his partner. The social worker should FIRST consider that sexual dysfunction:

A. Illustrates signs of relapse. B. Can occur early on in recovery. C. Is uncommon and requires medical attention. D. Is the result of the client's age.

A social worker uses a standard progress note to document treatment sessions. After a recent session with a client, the social worker realizes that she left out a key piece of information related to the client's plan before the next session. In order to document this additional information, the social worker should:

A. Include it at the bottom of the note with the other pieces of planning information B. Make an addendum to the previous note and sign and date it C. Wait until the next session and note it as part of the weekly assessment D. Destroy the original note and create a new note with the updated information

6-year-old and 4-year-old siblings were placed in foster care after reports of neglect were made by a neighbor. Despite ongoing casework and an extension of services, the single mother of the young kids has made no progress in changing her behaviors. It has been determined by the child services agency that the process to terminate parental rights should begin to take place. The NEXT step the foster care worker should take is to:

A. Increase meetings with the mother to encourage a change in behavior. B. Appoint a legal guardian for the children. C. Meet with the children in order to prepare them for changes that will occur as parental rights are terminated. D. Advocate for the mother by encouraging Child Protective Services (CPS) to give her another extension for casework services.

A couple in their early 40's seek help from a social service agency after adopting two children, ages 8 and 12. During the assessment, the parents report the kids are "out of control" and they can't "get them to do anything." When the social worker clarifies what the parents mean, they report that the kids don't get themselves ready for school and haven't been completing homework on time. They go on to report that they themselves go to bed at 10pm, but the children are often still up, which makes it harder for them to get up in the morning. The social worker should:

A. Make a report to CPS regarding neglect B. Provide a referral for a parenting skills class C. Assist the parents in identifying ways to establish basic routines with the children. D. ​​​​​​​Meet with the children to obtain information from their perspectives

A social worker at a residential treatment center is having trouble focusing at work due to a recent divorce. A co-worker tells the social worker that a few patients have come up to her saying they are frustrated by the social worker's negative demeanor. The social worker should FIRST:

A. Meet individually with the patients who are frustrated. B. Decrease her caseload. C. Consult with a supervisor. D. Be more conscious of her demeanor around patients.

A social worker was recently assigned as case manager for a family with three children who are in foster care due to neglect by their mother. While developing service plans for the siblings, the social worker should recognize that it is MOST important to:

A. Meet with the foster parents' to address their concerns. B. Use a solution-focused intervention to address the individual needs of each child by creating separate treatment goals. C. Develop the same treatment goals for all three siblings. D. Develop treatment goals around the reason that the children were removed from their home.

A school social worker meets with a sixth grade teacher regarding a student who was recently diagnosed with Type I diabetes. The teacher reports that the student has isolated herself from the other students, has fallen behind, and isn't interested in participating in class like she used to be. The social worker FIRST:

A. Meet with the student and explore her feelings about receiving the diagnosis. B. Contact the student's parents to obtain additional information. C. Help the teacher understand the effects of being diagnosed with a long-term illnessduring this critical stage of identity development. D. Refer the student to a support group.

While working at a residential treatment center, a social worker overhears a recently terminated client ask another social worker to meet at a local bar for dinner. The social worker agrees saying that she is looking forward to celebrating the client's completion of the program. The social worker in this situation should FIRST:

A. Meet with the supervisor and colleague to discuss the issue. B. Address the issue with a supervisor. C. Address the issue directly with the colleague. D. Report the colleague to the licensing board.

Parents of a 4-year-old boy seek help from a social worker in private practice after finding their son dressing up in his mother's skirts and high heels. The parents report that when they asked their son what he was doing, he said, "going to work, like Mommy!" The social worker should:

A. Normalize the parents' concerns B. Explain that these are early signs that their son may be transgender C. Complete a biopsychosocial assessment D. Provide the parents with psychoeducation around typical child development to normalize the child's behavior

A social worker in a small rural community has decided to consult with a colleague about a client who does not seem to be progressing in therapy. As part of the consultation the social worker's BEST course of action is to:

A. Obtain a release from the client since the colleague might know him / her. B. Make the best effort at omitting any identifying information about the client. C. Disguise the client's identity by changing demographic information. D. Convey minimum amount of information needed to get help.

A therapist is treating a court-mandated client with a substance abuse problem. The therapist has decided to utilize motivational interviewing for the treatment of the client's substance abuse. Which of the following techniques are NOT considered part of motivational interviewing?

A. Open-ended questions B. Confrontation C. Reflective listening D. Summaries

A social worker in a nursing home is meeting with a new client, an 82-year-old woman. As part of the assessment, the social worker administers a mental status exam to get a baseline measure of the client's cognitive functions. All of the following would be part of the mental status exam EXCEPT:

A. Orientation to day and time B. Appearance C. Ability to interpret a common proverb D. Attachment history

A 20-year-old student presents for therapy to address feelings of anxiety and confusion. He indicates that he is in an intimate relationship with another male student, and his parents are coming to visit this weekend. He states, "I don't know what to do. They will disown me if they find out." What interventions are indicated to treat this client?

A. Present a nonjudgmental stance regarding the student's sexual orientation and encourage the student to be open with his parents. B. Assist the student in problem-solving around the immediate concern and work over the longer term to explore issues of sexual identity. C. Normalize the student's internal conflict regarding sexual dysphoria and develop a plan to safely meet with his parents. E. Schedule a family therapy session and allow disclosure of sexual orientation in a safe and supportive environment.

A client reports that she recently awoke in the middle of the night to sounds of crying children in the apartment next door. She went to see if everyone was okay and discovered that the parents had left the 8-year-old, 5-year-old, and 2-year-old home alone. The client suspects that the parents are using drugs and expresses a desire to help the kids. The social worker should FIRST:

A. Process client's feeling about the situation B. Discuss ways the client could help the kids C. Inform the client that the social worker must file a child abuse report D. Determine if the children are in danger

A social worker has been treating a client who has a pattern of late payments for services. The social worker recently sent out three written requests for payments that are overdue, which the client has disregarded. At the next session, the social worker has decided to tell the client that they will need to begin termination due to the lack of payments. Before proceeding with termination, the social worker should FIRST:

A. Provide the client with a different date or time that is easier for her to attend. B. Explore the unconscious meaning of the nonpayment with the client. C. Provide the client with referrals to other treatment providers. D. Ensure that the client understands the payment for service agreement.

A 25-year-old client has been working with a social worker in a community mental health clinic. The client is often late for appointments and states that she forgets to call in advance to cancel despite being told the attendance policy. When the social worker confronts the client she states, "What's the big deal? It's too hard to be here all the time and you are being unrealistic." The MOST appropriate action for the social worker to make is to:

A. Provide the client with a referral to another therapist. B. Reiterate the attendance policy to the client. C. Provide the client with an alternative appointment time. D. Facilitate a conversation about the client's feelings and view of the problem.

A social worker sees a new client referred because of heavy alcohol use. During the initial assessment the client questions the social worker's credentials and competence. What should the social worker do FIRST?

A. Provide the client with honest information regarding social worker's credentials. B. Confront the client about the evasive responses. C. Ask how the client feels about seeking help. D. Explain that the social worker has treated a lot of patients like him.

A social worker who is new to the field is meeting with a client for the first time to evaluate what services would be appropriate for her. The social worker thinks that the client may have obsessive compulsive disorder. What should the social worker do FIRST to locate appropriate services?

A. Provide the client with referrals to a mental health clinic. B. Consult with colleagues who are knowledgeable about resources. C. Research effective treatments for OCD. D. Refer the client to a psychiatrist.

A social worker has started to work with a 15-year-old boy who came in feeling confused because he has started to have feelings for other boys at school. The client states that his parents are very religious and would be upset if they found out he was gay. The social worker realizes after meeting with the client for a month that her own values may be interfering with her relationship with the client. The MOST appropriate action for the social worker to take is to:

A. Provide the client with referrals to another social worker. B. Meet with the client and his parents in order to address the client's concerns. C. Continue to meet with the client. D. Meet with a supervisor to address the value conflict.

During a session with a client who is diagnosed with anxiety, the social worker notices the client has stereotyped beliefs against various racial groups. These beliefs go against the social worker's personal values. The social worker should:

A. Provide the client with referrals to another therapist. B. Address the client's stereotyped beliefs in a nonjudgmental manner. C. Address the presenting problem while respecting the client's beliefs. D. Discuss the conflict in value system with the client.

A 20-year-old woman recently started therapy with a social worker due to posttraumatic stress symptoms after being sexually assaulted on her college campus. The mother of the client has contacted the social worker asking for information regarding the client's sessions saying that she " wants to stay in the loop." The social worker's FIRST response to this request would be to:

A. Provide the mother with minimal details of the client's treatment. B. Encourage the mother to speak with her daughter directly. C. Set up a family session where the mother is able to directly address her concerns with her daughter. D. Decline to provide information by addressing the confidential nature of the therapeutic relationship with the mother.

The parents of a 2-year-old meet with a social worker stating that they are frustrated with their son who is constantly refusing to do what they say. The mother states, "All he does is say no, or does the opposite of what I want him to do." The father admits that his wife and him have minimal understanding of child development and what behaviors a 2-year-old should be exhibiting. What should the social worker do FIRST to help serve this family most effectively?

A. Provide the parents with a referral to a parenting class. B. Empathize with the parents' frustrations and address ways they can better manage their child's behaviors. C. Encourage the parents to read child development books. D. Complete a comprehensive family history.

A social worker is assessing a 2-year-old foster child. The social worker notes that the child cries when the caregiver leaves the room and the child is unable to be soothed upon the caregiver's return. What theory is guiding the social worker's assessment?

A. Psychodynamic B. Systems C. Behavioral D. Attachment

A social worker reads about furniture that is for sale in the newspaper. The social worker responds to the ad and discovers that a former client owns the furniture. The former client tells the social worker that she can have the furniture for 75 percent off, stating that she appreciated the social worker's previous help. The social worker's BEST response is to:

A. Purchase the furniture at the reduced price. B. Ask a third party to follow through with the purchase. C. Allow the client to decide how much she wants to sell it for. D. Purchase the furniture at the originally advertised price.

A researcher with a PhD in social work is interested in looking at help-seeking behaviors in her community. She plans to distribute a survey to all registered voters and analyze the results. This study is best described as using:

A. Quasi-experimental design B. Single subject design C. Cross-sequential design D. Cross-sectional design

A social worker is asked to assist an elderly client in making alternative living arrangements. In the initial interview, the client repeatedly attempts to discuss past events. What is the social worker's MOST appropriate initial response to the client?

A. Redirect client to the housing issue. B. Facilitate discussion of the recollections. C. Refer client to a support group for elders. D. Ignore references to the past.

During an initial meeting, a 23-year-old female client complains of depression. The client reports a history of regular cocaine use, although she reports refraining from use for several days and a desire to maintain sobriety. The social worker should FIRST:

A. Refer client for a psychiatric evaluation. B. Recommend the client attends NA groups in conjunction with individual therapy. C. Explain to client that depression is a common withdrawal symptom. D. Explore client's level of motivation to change and maintain sobriety.

A school social worker is a member of the local school board and believes extracurricular activity policies are discriminatory toward students who come from lower-income families. What should the social worker do FIRST?

A. Refer families to lawyers who specialize in discrimination B. Bring concerns to the next school board meeting C. Utilize local media to inform the community of these concerns D. Document instances of discrimination

A client who has been in treatment for five months for Major Depression recently lost her job and tells you that she can no longer afford her full co-payment but wants to continue therapy. The social worker should:

A. Refer her to get unemployment benefits. B. Reduce her co-payment. C. Consider reducing co-payment temporarily, but inform the insurance if done. D. Refer her to a low-fee community mental health center.

A Native American man seeks therapy for alcohol abuse. In developing a treatment plan, how should the social worker take into consideration both his culture and his struggle with alcoholism?

A. Refer him to an inpatient substance abuse facility. B. Ask how many friends in the Native American community he has. C. Recommend that he enroll in a group for Native Americans dealing with alcoholism. D. Consult his tribe in regard to how to treat this issue.

A social worker meets with a client who was recently diagnosed with Schizophrenia. The client reports that her psychiatrist has told her that medication is necessary to manage her symptoms but that she does not want to be on medication for the rest of her life. She states that she hasn't told anyone about the diagnosis and that, "everyone will just think I'm crazy, because I am." The social worker should FIRST:

A. Refer the client back to her psychiatrist to discuss her feelings about medication B. Explore the client's thoughts and feelings about her diagnosis and prognosis C. Refer the client to a support group for people with Schizophrenia D. Ask the client to sign releases so the social worker can contact her family members

During a supervision session at a community agency, a social work supervisor makes a sexual advance towards their clinical supervisee. What should the social work supervisee's INITIAL action be?:

A. Report this incident to HR at their agency B. Begin working toward obtaining a new supervisor C. Explain to the supervisor that this is not appropriate D. Report the supervisor to their regulatory board

A hospital social worker is working with a 35-year-old client who was involuntarily committed to the psychiatric unit due to the risk of self-harm. The client has a long-term history of anorexia nervosa and, as a result, is experiencing many medical complications. While meeting with the social worker, the client states that she does not want to be in the hospital, and has been refusing the hospital food. The client asks to be referred to a day treatment program. What should the social worker do FIRST?

A. Refer the client for a medication evaluation. B. Remind the client that her hospitalization is not voluntary. C. Support the client in her efforts to be discharged. D. Point out the impact of the client's behaviors on her overall health.

A hospital social worker is developing an aftercare plan for a 75-year-old client who is confined to a wheelchair after a car accident. The client wishes to relocate to an area with limited support services, despite needing weekly medical attention. She currently lives with her daughter who is unhappy with her decision to relocate. The client has asked for help from the social worker during this transition. The hospital social worker should FIRST:

A. Refer the client to agencies in the area where she wishes to relocate. B. Meet with the medical team to gain a better understanding of the client's medical needs. C. Conduct a meeting with the client and her daughter to understand both their concerns. D. Encourage the client's self-determination by creating a discharge plan that would help the client transition to the new location smoothly.

A social worker has been seeing a client for 3 months when she discovers that both she and the client volunteer for the same homeless organization. The client, who recently moved to the area from another state, asks if they can volunteer together at an upcoming event because she doesn't know anyone in the area yet. The social worker should NEXT:

A. Refer the client to another social worker because of the dual relationship. B. Help link the client to other volunteers for social support. C. Deny the request. D. Address the dual relationship with the client and set appropriate boundaries.

A 45-year-old female has a history of paranoid delusions and crack cocaine use. She is distrustful of the system and service providers in general. She has refused medication and substance abuse treatment. What INITIAL clinical strategy should the social worker use in this case?

A. Refer the client to substance abuse treatment. B. Attempt to establish a trusting relationship to engage the treatment. C. Assist the client in seeing a psychiatrist to treat her mental illness. D. Provide psychoeducation on the relationship of drug abuse.

A 72-year-old Mexican-American man has been referred to the social worker by his adult day care facility. The man reports that his wife recently died after a long battle with cancer and that he has been working on an altar in his house to honor her. He reported to his friends that he can't wait to see his wife's ghost and regularly leaves flowers and gifts on the altar for her. The social worker should FIRST:

A. Refer the man to his primary care physician for a medical evaluation B. Assess the client for other signs of psychosis C. Explore the client's cultural beliefs around death and dying D. Refer the client to a grief support group

A social worker is meeting with the parents of a 7-year-old boy who was recently diagnosed with ADHD. The parents met with a psychiatrist who recommended medication and behavioral management therapy. The parents are resistant to medication. What should the therapist do first?

A. Refer them back to the psychiatrist. B. Explore their concerns about medication. C. Begin behavioral modification training. D. Encourage them to try out medication before they judge it.

A gay couple seeks therapy for communication and commitment issues. During the initial session, one of the clients asks if the social worker believes in gay marriage. How should the social worker respond?

A. Refer them to a gay therapist. B. Explore why the information is important to the client. C. Share views of gay marriage. D. Ask how the couple feels about gay marriage.

A social worker is treating a client who recently informed her employer that she was five months pregnant and planning to take maternity leave following the birth of the child. The client soon after received written notice of termination from employment due to "personnel conflicts." The client asks the social worker what she should do. What is the BEST response the social worker should give?

A. Request disability insurance. B. Explore alternative employment options. C. Seek legal advice regarding the termination. D. Confront her manager about the termination.

While doing an intake assessment with a 22-year-old woman, the client reports to the social worker that she recently has been feeling sad and anxious due to recent family problems. She also states that before coming to meet the social worker she took an entire bottle of Advil, saying that she wants to kill herself. The social worker is unable to get in touch with the client's emergency contact. The social worker should FIRST:

A. Request for the client's consent to call 911. B. Finish the assessment and then call 911. C. Call 911 immediately. D. Bring the client to the nearest hospital.

A 22-year-old man who is diagnosed with Borderline Personality Disorder meets with a social worker. During the initial interview with the social worker, the client states the he feels like he is always in crisis. The social worker feels uncomfortable working with the client since she has no experience in this area. Social work ethics require that the social worker:

A. Research effective treatments for borderline personality disorder. B. Consult with a colleague who is skilled in treatment of borderline personality disorder. C. Provide the client with a referral to a therapist who is knowledgeable in this area. D. Assist the client in identifying effective coping mechanisms.

A school social worker meets with a 15-year-old student who was sent to the social worker due to skipping classes and declining grades in the past two months. The social worker notes that the student has never had behavioral problems, and up until this point has been a good student. With this information, the social worker should FIRST:

A. Schedule a meeting with the child's parents. B. Refer the child to an individual therapist. C. Assess the child for learning disabilities. D. Meet with the student to explore the events leading up to these changes.

A mother and her 8-year-old daughter are referred to a social worker after a school counselor receives several reports that the girl has been bullying classmates. The mother is confused and says that her daughter gets along well with her two siblings and other children in their apartment complex. After completing a social and developmental history from the mother, the social worker should NEXT:

A. Schedule an individual session with the girl. B. Have the mother sign a release to talk with the school counselor. C. Schedule a session with the siblings to gather additional information. D. Request the girl's school record.

A social worker meets with the director of a community mental health center. During the meeting, the social worker conveys various frustrations about the agency. The BEST way for the director to address this is by:

A. Scheduling a meeting with the director, the social worker's supervisor, and the social worker to address the concerns. B. Scheduling a meeting with the social worker's supervisor to address the concerns. C. Listening to the concerns and encouraging the social worker to speak with a supervisor. D. Listening to the concerns and assessing the best way to create change based on the social worker's concerns.

A social worker at an agency received a written request about a former client to send a summary of client's mental health treatment. What is the BEST response to the request?

A. Send all treatment records. B. Send only medication. C. Send only progress. D. Send only selected portion of records.

A social worker has many clients who live in residential group homes in a city. During recent meetings with clients, the social worker notices that the group homes are providing inadequate treatment. What should the social worker do FIRST to try to correct the problem?

A. Set up a meeting with families of residents to encourage change. B. Create more appropriate outpatient programs to discourage clients from needing higher levels of care. C. Facilitate the creation of interest groups in which different options can be explored. D. Meet with city officials to discuss the need for evidence-based treatments.

The director of a social services agency is interested in developing a new social skills program to help children with ADHD. In working with staff, the director splits the appropriate clients into two groups: one group will undergo the experimental social skills group; the other will participate in an evidenced-based social skills protocol. This is best described as:

A. Single-subject design B. Quasi-experimental design C. Randomized controlled trial D. Cross-sectional design

A 31-year-old woman is admitted to the hospital following a suicide attempt. During the intake interview, the woman reveals that she was sexually abused by an older cousin when she was 10. The social worker should NEXT:

A. Speak with a family member to determine the truth of the abuse. B. Explore the ways in which the abuse contributes to the client's presenting problem. C. Explore the specifics of the abuse with the client. D. Complete a comprehensive sexual history.

A social worker is performing an intake assessment with a client who is HIV positive. During the interview, the client admits that he does not inform sexual partners of his diagnosis, stating that he does not think it is their business. The social worker should FIRST:

A. Speak with the client's partners regarding his HIV status. B. Encourage the client to speak with his sexual partners regarding his HIV status. C. Refer the client to a support group for HIV positive individuals. D. Speak with the local public health authority to address the issue.

When writing case notes for a court-mandated client, the social worker should include information related to the client's:

A. Specific treatment needs. B. Level of participation in treatment. C. Improvement in use of coping mechanisms as needed. D. Progression toward specific goals and objectives as stated on the treatment plan.

What co-occurring condition is MOST likely to be diagnosed in a client with Schizophrenia:

A. Substance Use Disorder B. Anxiety Disorder C. Depressive Disorder D. Personality Disorder

A social worker receives a referral for a 32-year-old woman who has a history of depression, low self-esteem, and suicidal ideation. During the initial session, the social worker notices that the client fails to make eye contact, speaks softly, and often appears to be lost in thought, going for long periods of time without speaking. The social worker should FIRST assess for:

A. Substance abuse B. An autism spectrum disorder C. A personality disorder D. A history of trauma

Which of the following treatments would be MOST appropriate for a teenager who experienced childhood sexual abuse?:

A. TF-CBT B. Psychodynamic Therapy C. Motivational Interviewing D. DBT

A new client with a history of depression started taking Paxil a week ago. The client reports frustration and despair that she isn't feeling better. The social worker should?

A. Tell the client to increase the dosage. B. Refer the client to a new psychiatrist. C. Encourage the client to try a new medication. D. Inform client that Paxil can take 2-6 weeks to have impact.

A Hmong couple is referred to the school social worker. Their 6-year-old daughter has been sick recently and the teacher is concerned that the parents are not getting adequate health care. The parents share that they immigrated here five years ago. The social worker knows very little about the Hmong culture. The social worker should FIRST:

A. Tell the couple that she knows very little about Hmong people. B. Research Hmong cultural beliefs. C. Explore their cultural beliefs and how it may be impacting the current situation. D. Consult with a social worker who is an expert on the Hmong culture.

A supervisee is unexpectedly dismissed from her job by her supervisor. The supervisee requests that she wants to contact her client about her dismissal but her supervisor denies her request. The supervisee's BEST action is to:

A. Tell the supervisor that the code of ethics states proper termination B. Call state board and report supervisor C. Send letters to clients D. Appeal supervisor's decision to managed care

A social worker has referred a client to a therapy group for men with depression. The social worker speaks with the support group leader to determine if the group will be a good fit for the client. During the conversation the group leader asks for personal information about the client including if he has a history of hospitalizations or abuse in order to ensure that the client is a good fit for the group. In this situation, the social worker should proceed by FIRST:

A. Telling the group leader that the client needs to be consulted first before that information can be provided. B. Releasing as much information as possible to ensure that the client is a good fit for the group. C. Stating that information is not relevant as to whether the client will be a good fit or not. D. Requesting information about the other clients in the group.

A client receives treatment for substance use by a social worker who is a recovering alcoholic. During the course of this client's treatment, the social worker begins to experience cravings to use alcohol and feels increasingly angry toward the client. Which of the following actions should the social worker take?

A. Terminate services with the client and provide appropriate referrals. B. Seek immediate consultation with a supervisor for identifying and resolving countertransference issues. C. Return to alcohol treatment program for personal assistance with maintaining sobriety. D. Share feelings with client as way of creating a supportive common ground of experience.

A hospital social worker is preparing a discharge plan for an 82-year-old woman who fell in her home and broke her hip. The client previously lived at home alone and would like to return home if at all possible. In developing a discharge plan, the social worker should FIRST assess:

A. The client's social and familial supports B. The client's commitment to participate in physical therapy C. The client's cognitive functioning D. The client's ability to independently engage in activities of daily living

A social worker is interviewing the 19-year-old single mother of a 5-year-old girl after receiving reports of alleged child abuse from a teacher. The social worker learns that the mother also has previous substantiated allegations of child abuse as well. The factor that carries the MOST weight in the assessment of abuse is:

A. The lack of involvement from the child's father. B. The previous substantiated report of child abuse. C. The age of the child's mother. D. The amount of family support the mother has.

A social worker is evaluating an 8-year-old child of a different ethnic background whose mother was recently reported to child protective services due to alleged child abuse. When assessing the risk factors, the social worker must FIRST:

A. Understand culturally appropriate body language and communication. B. Understand how one's own cultural values and beliefs influence the social worker's ability to deal with issues regarding diversity. C. Understand that the clinical interview is more important than standardized assessments. D. Use a solution-focused model to develop effective treatment goals.

A 14-year-old student is referred to a social worker due to recent behavior changes at home and in school. In order to develop a therapeutic relationship with this client, the social worker should FIRST:

A. Validate the client's feelings and address his recent behavior changes. B. Explore precipitating events to the behavior changes. C. Address confidentiality with the client. D. Explore the client's feelings regarding coming to therapy.

A 7-year-old child who was sexually abused by a family friend is being treated by a social worker. During one session, the client's mother reports that his teacher contacted her to report that the child was found touching the genitals of another child on the playground. The mother reports that she is concerned by this behavior and unsure what to do. The social worker should explain that this behavior:

A. Will almost certainly decrease over time. B. Probably means the child is still being abused. C. Necessitates continued treatment. D. Substantiates that the child was actually abused.

A social worker has been assisting a 75-year-old woman who recently had surgery and is now on bed rest. A neighbor has been helping the client with everyday tasks so that she can remain at home. During the most recent visit, the social worker has become concerned that the neighbor is stealing valuable items from the client and is not providing appropriate care. The client reports that she wants to continue to live at home, but the home health staff is concerned that intervening will damage the relationship with the client. The social worker's FIRST responsibility is to:

A. Work with the client to develop an alternative treatment plan. B. Address concerns with the neighbor. C. Encourage the client to confront her neighbor. D. Report the neighbor to Adult Protective Services (APS).

A social worker has been seeing a client for several months. The client reports feeling depressed, overwhelmed, and hopeless about the future. The social worker should do all of the following EXCEPT:

A. collaborate with the client to develop goals for behavior change. B. address client's dysfunctional thinking patterns. C. accept the client's views of self and circumstances. D. ask questions to identify client strengths and successes.

A 42-year-old woman is in short-term treatment with a social worker for issues in her long-term relationship. In session, the client frequently deflects questions, makes small talk, and focuses on issues in her best friend's marriage. As the social worker becomes aware of this dynamic, the MOST APPROPRIATE intervention would be to:

A. explore how the client deals with problems outside of session. B. respect the client's style of relating. C. confront the client with regards to this pattern. D. remain silent and allow the client to bring up the material.

A social worker meets a client who is in her 8th month of her first pregnancy. The client and social worker have been working for several years around the client's Generalized Anxiety Disorder. During the session, the client discloses that she has continued to take her PRN Xanax even though it may have a harmful impact on her baby. She confirms that her OB-GYN is aware of this and has provided her with the relevant research. The social worker realizes she is feeling judgmental about the client's ongoing medication use. The BEST approach to this situation is:

A. express a genuine sense of being perplexed by the client's decision. B. contact the psychiatrist and share concerns. C. have another conversation with the client making sure she is adequately processing the reality of negative side effects. D. manage countertransference by consulting with a colleague.

A social worker meets with a single parent struggling to provide adequate housing, food, and health care for himself and his two younger children. The FIRST thing the social worker should do is:

A. help client develop coping skills to deal with stressors. B. file a report regarding possible neglect. C. provide client with information about financial services. D. provide client with referrals for services that meet his specific needs.

A 32-year-old male with criminal history has been in therapy for several months to deal with anxiety and relationship issues. One session he shares that he robbed an electronics store three days prior. The social worker's BEST response is to:

A. make a police report. B. call the store and inform them of what happened. C. talk to the client about turning himself in. D. maintain confidentiality and process the incident with the client.

A social worker in a long-term care facility is meeting with a new resident who has recently started showing signs of dementia. The social worker administers the Mini Mental Status Exam as part of the intake and will repeat this measure on a weekly basis. The BEST reason for including this measure is to:

A. ​​Ensure the client's cognitive functioning does not decline B. Document the effectiveness of treatment C. Track changes in the client's cognitive functioning over time D. Justify the appropriate level of care.

A 35 year-old African American woman presents for an initial appointment with a white female social worker. The client has worked at a predominantly male technology company since obtaining her doctoral degree, but is experiencing increasing frustration and anger about how she is treated in the workplace. The social worker says, "I can absolutely imagine how frustrated you are. I'm sure your coworkers aren't intentionally leaving you out, but that's what's happening." The client says angrily, "You absolutely CANNOT imagine my frustration! You have no idea what you're talking about." The social worker should next:

A. ​​​​​​​Explain that she was trying to empathize with the client B. ​​​​​​​Offer to make a referral to a Black social worker C. Explore how this behavior relates to her difficulties at work D. Acknowledge the social worker's misstep and the client's real experiences of discrimination

(Correct) The answer to question 101 is C: Rationale:

According to the Code of Ethics, if a co-payment is reduced the insurance company needs to be made aware. Therefore, since the client has already been seeing the social worker for 5 months, it would be most beneficial for the client to continue to see if the social worker at a reduced co-pay fee (C) in order to avoid a disruption in the client's treatment.

(Correct) The answer to question 140 is C: Rationale:

Accepting the client's view of hopelessness would not be appropriate (C). Collaborating with the client on goal development (A), addressing dysfunctional thinking patterns leading to the client's view (B), and identifying client's strengths (D) would all be fitting interventions.

(Wrong) The answer to question 70 is D: Rationale:

All of the answers here are reasonable things to assess, but the most important thing for the social worker to determine is the client's ability to engage independently in daily self-care. Whether or not the client can reasonably care for herself is the first thing to determine; everything else would go from there.

(Correct) The answer to question 49 is A: Rationale:

An example of a question where you have to choose the best of the worst. Start by eliminating bad answer. Both B and D break the client's confidentiality by filing a police report, thus they are out. Answer C implies that the victim is responsible for the rape, which is wrong. The only answer option left is A. The word "instruct" is rather strong, yet it is the only option that doesn't break confidentiality, and refers the client to emergency room where she should receive medical attention.

(Wrong) The answer to question 7 is A: Rationale:

Answer (A) is the best place to start. If the supervisor refused, the social worker could do (B). (C) is a bad choice because it is going directly against the supervisors wishes. (D) is wrong because there is no indication that managed care is involved nor would they be the ones to address this issue.

(Wrong) The answer to question 12 is B: Rationale:

Answer (B) is correct. Self-reports, usually administered in a written questionnaire, have been shown to give the most accurate information regarding eating disorders.

(Wrong) The answer to question 10 is C: Rationale:

Answer (C) is the best answer because it addresses the client's concern. You could then do (D). Answer (A) is inaccurate as bereavement varies from person to person and (B) is too general and doesn't address the client's concern.

(Correct) The answer to question 119 is A: Rationale:

As social workers it is important to be aware of our own personal cultural beliefs and how they may affect our clients. Therefore, the best way for the social worker to become competent in multicultural practices is by not only gaining an understanding of diverse client's, but one's own beliefs and prejudices as well (A).

(Correct) The answer to question 124 is A: Rationale:

As social workers, if we are aware of unethical behavior, we should address it directly first before going to the board; in this case, the proposed study is unethical because it withholds treatment from those in need. B would also be unethical. C is a potential action, but should be done after the ethical issue is resolved. D could be done after A if the agency decides to proceed with the study.

(Correct) The answer to question 85 is B: Rationale:

B is correct because all of the elements listed are aspects of cognitive therapy. Cognitive therapy is a collaborative and goal-oriented therapy that explores patterns of thinking and beliefs that lead to self-destructive behaviors. Once a person understands the relationship between thoughts and behaviors they can begin to modify these patterns and cope with stressors in a more positive manner.

(Correct) The answer to question 121 is B: Rationale:

B is correct because discussion of recollections, sometimes called reminiscence therapy or memory care, is really important for the psychological well-being of older adults. Therefore, the social worker should facilitate discussion around these memories.

(Wrong) The answer to question 149 is B: Rationale:

B is correct because the accommodation process is when one's schemas or ideas are altered as a result of new information or new experiences. In this case, the boy's college experience is allowing him to be surrounded by individuals from a group that he previously stereotyped. Through this experience he realizes that the information he learned from his family was wrong.

(Correct) The answer to question 129 is B: Rationale:

B is correct because the best way to address staff productivity is by exploring the needs of staff members in a small group where everyone is able to voice their needs and motives (B). A, C, and D are incorrect because none of those options give staff the ability to express their own needs.

(Correct) The answer to question 38 is B: Rationale:

B is correct because the client has already met the agreed-upon goals. Therefore, issues around termination should be addressed while decreasing the frequency of sessions to slowly help the client adjust to her life without therapy. A is incorrect because the client has already met her treatment goals. C and D are unnecessary; there is no need to create new goals for the client.

(Correct) The answer to question 41 is B: Rationale:

B is correct because the client reports fatigue and difficulties in sleep and eating. Therefore it is most important for the social worker to assess the client's health and ability to provide basic care during an initial evaluation. A, C, and D, are all important and should be evaluated but the client's presenting health problems need to FIRST be assessed.

(Wrong) The answer to question 77 is B: Rationale:

B is correct because the social worker should advocate for the rights of the client by meeting with the principal who has authority over the school administration. A is incorrect because there is no need to involve the parents yet. C may be necessary at a later date, but should not be done first. D is unnecessary to do.

(Correct) The answer to question 47 is B: Rationale:

B is the best answer as it respects the client's style of communicating and meets the client where she is. A is not the best answer because it may be disruptive and it is often difficult to find a therapist who shares an identical "culture" with the client. C is inappropriate since the client is not avoiding answering, but is answering in a different way. D would not be helpful as it disrespects the client's communication style and may hurt therapeutic rapport.

(Wrong) The answer to question 24 is B: Rationale:

B is the best answer because decongestants often contain pseudoephedrine, which can trigger hypomanic and manic episodes. A is not indicated in the question; C would not the be the case for the general population ; D does not make sense given the information provided.

(Correct) The answer to question 132 is B: Rationale:

B is the best answer because it is the most effective way to improve the chances of success in this treatment relationship. A, C, and D may all be part of the treatment planning process, but would not be done before the therapist understands the barriers to previous attempts at treatment.

(Correct) The answer to question 164 is B: Rationale:

B is the best answer because it's important for the social worker to explore the parents' concerns about medication and what is behind that concern before further steps are taken.

(Correct) The answer to question 33 is B: Rationale:

B is the best answer because the client is exhibiting signs and symptoms of potential early childhood trauma. A is not indicated and is not related to trauma-informed care. C and D might be useful interventions, but given the client's presentation, B should come first.

(Wrong) The answer to question 147 is B: Rationale:

B is the best answer because the client is on an involuntary hold and has few options at this time. A may be needed, but is likely already part of the treatment protocol at the hospital. C is not the best answer because the client appears to be at risk of self-harm. D might be a good option, but not before discussing the nature of the hospitalization.

(Wrong) The answer to question 82 is B: Rationale:

B is the best answer because the client needs immediate assistance in figuring out how to manage his parents' visit, but is also expressing feelings of confusion about his sexuality that should be addressed over future sessions. A is not the best answer because it may not be safe for him to disclose his current relationship to his parents; his fear may be legitimate. C is not the best answer because it doesn't address his internal conflict as strongly as B; normalizing his feelings might be immediately helpful, but does not address the longer-term issue. D is not the best answer, again because the client has stated that he does not feel safe disclosing his relationship, and the social worker should not pressure him into doing this.

(Correct) The answer to question 106 is B: Rationale:

B is the best answer because the social worker needs to understand WHAT the client is thinking about before moving forward. If the thoughts are psychotic in nature, the social worker might proceed with a medication referral (A). C and D could be done at a later point in time, but are not indicated before the social worker understands the client's thoughts more thoroughly.

(Wrong) The answer to question 163 is B: Rationale:

B is the best answer because the wife will be able to provide information necessary for the assessment. A is not the best answer because the client has already been identified as having a stroke; a mental status exam is not likely to help the social worker assess the client at this point. C is not the best answer because it jumps to a conclusion when the effects of the stroke may only be temporary. D is unnecessary since the wife is already present and participating in the interaction.

(Correct) The answer to question 86 is B: Rationale:

B is the best answer because this appears to be a school-based issue and the collateral information from the school is necessary for assessment. A and C might both be good steps after the collateral information from the school is obtained. D is not the best answer because reviewing the school records will not provide as much information as contact with the counselor.

(Wrong) The answer to question 50 is B: Rationale:

B is the best answer in this case because the social worker needs to understand more about the student's overall functioning and adjustment to the diagnosis; A and D might be useful, but not until after the social worker contacts the parents for more information. C will likely not affect the student's behaviors.

(Correct) The answer to question 79 is B: Rationale:

B is the best answer. When a previously existing note needs to be updated/modified, the proper steps to take are to make an addendum to the note, signing and dating it.

(Wrong) The answer to question 26 is B: Rationale:

B is the correct answer because motivational interviewing is non-judgmental and non-confrontational. Motivational interviewing tries increase a client's awareness of a problem in a warm and empathic therapeutic setting by using open ended questions (A), reflective listening (C), and summary statements (D).

(Wrong) The answer to question 81 is B: Rationale:

B is the correct answer because participants have been knowingly placed in the experimental vs. control group; while the participants themselves may not know which group is which, the researchers do, which makes this quasi-experimental.

(Correct) The answer to question 30 is B: Rationale:

By self-disclosing the social worker's own loss, she allows the group to learn that she shares an understanding of their experience (B). D would be inappropriate as the social worker is not in a role to receive support from the group. The social worker's personal experience does not necessarily establish her as an effective group leader (A) or show that she understands the effects of additions (C).

(Wrong) The answer to question 148 is C: Rationale:

C describes all important aspects of Strategic Family Therapy, including the use of directives that help alter patterns of communication between family members, restraining and positioning strategies, and modeling/educating about communication skills. In Strategic Family Therapy, the therapist joins the family system and focuses on the process instead of the content.

(Correct) The answer to question 157 is C: Rationale:

C describes the most appropriate way to terminate with the client, which includes sending documentation of termination and providing referrals for continuity of care. A is not the best option because someone else could hear the message, violating confidentiality. B does not tell us how termination will take place and also does not provide referrals for ongoing treatment. D is breaking confidentiality, which is unwarranted, as this does not constitute an emergency.

(Correct) The answer to question 76 is C: Rationale:

C is correct because it allows for the client to be in a group with others who understand his culture and his addiction. A is incorrect because nothing in the stems leads us to believe he needs a higher level of care. B is incorrect because it does not necessarily take into account his addiction. D is incorrect because it would be a breach of confidentiality for the social worker to address his tribe.

(Correct) The answer to question 61 is C: Rationale:

C is correct because solution-focused treatment is a brief, goal-directed therapy where solutions are focused on rather than problems. With that in mind, it would be most appropriate for the social worker and client to collaboratively work together towards a solution.

(Correct) The answer to question 115 is C: Rationale:

C is correct because some symptoms of reactive attachment disorder include markedly disturbed and developmentally inappropriate social relationships, aggressive behaviors towards peers, and difficulty being soothed. This is seen in the stem when the mother describes the child preferring to be with strangers over her and her husband, disengagement with other kids his age, and a withdrawn appearance. The stem also gives the clue that the child was severely neglected as an infant and therefore never had secure attachment figures.

(Correct) The answer to question 73 is C: Rationale:

C is correct because the client it would be inappropriate for the client to be seen while he is in an intoxicated state. Therefore, the social worker should reschedule the session and help the client find a way home safely.

(Wrong) The answer to question 144 is C: Rationale:

C is correct because the social worker needs to assess if potential abuse or domestic violence is occurring in the family. If abuse is occurring the social worker needs to meet with each family member when the father is not in the room so that they are free to express what is going on at home without fear of repercussions from their father (C). A would only be done later if the social worker assessed that abuse is occurring in the family. B and D may be assessed AFTER potential abuse has been explored.

(Wrong) The answer to question 51 is C: Rationale:

C is correct because the social worker should FIRST explore the family's cultural beliefs and the ways in which their beliefs impact their health care decisions. A would be inappropriate. B and D would be important to do only AFTER the social worker has explored the cultural beliefs with the parents.

(Wrong) The answer to question 2 is C: Rationale:

C is the best answer because by creating interest groups, the social worker can pull together like-minded individuals who also want change to occur. A, B, and D are all options that may be done at a later point but only after the creation of interest groups has occurred.

(Correct) The answer to question 1 is C: Rationale:

C is the best answer because it meets the parents where they are and assists them in developing some basic parenting skills that might help the family. A is not indicated in this situation. B and D are both decent interventions and could be done at some point, but would not be done before C.

(Wrong) The answer to question 138 is C: Rationale:

C is the best answer because it will lay the foundation for any step taken afterward in terms of teambuilding and ensure that team members can work together; A, B, and D do not address the task of creating a collaborative team.

(Wrong) The answer to question 94 is C: Rationale:

C is the best answer because it will make clear to the clients what information will be shared with the court and what is protected. A and B both may be true, but could differ depending on the specific arrangement between the court and therapist. D does not address the client's questions and may significantly decrease how effective the group could be.

(Correct) The answer to question 151 is C: Rationale:

C is the best answer because the MMSE is often used to track cognitive changes in older adults with dementia. A is not a good answer because administering the test will not prevent the client's symptoms from worsening. B is not the best reason for doing a MMSE. D may be a reason, but is not more important than being able to track changes in the client's functioning.

(Wrong) The answer to question 141 is C: Rationale:

C is the best answer because the behavior indicates that the child has not resolved sexual issues and needs continued intervention. A and B may not be at all true; D does not make sense since the validity of the abuse was not in question.

(Correct) The answer to question 162 is C: Rationale:

C is the best answer because the client experienced a traumatic event and is now exhibiting symptoms of PTSD: avoidance of the place/situation in which the trauma occurred, fear, and re-experiencing the event. A does not make sense because the traumatic event happened over 6 months ago; B does not make sense because the client does not report nightmares. D does not make sense because the client is not having issues with her memory.

(Correct) The answer to question 62 is C: Rationale:

C is the best answer because the client's behavior and statements are within cultural norms and the social worker should find out more about his cultural beliefs. A and D may be good interventions at some point, but not before the social worker meets the client where he is and understands more about his beliefs. B is too extreme in this situation.

(Wrong) The answer to question 75 is C: Rationale:

C is the best answer because the stem describes normal adolescent development. A may be true, but is not the reason for the daughter's behavior. B is not the best answer because there is nothing in the stem to indicate substance abuse. D is not the best answer because the stem does not mention noncompliance with meds, and while dietary changes may impact mood, they are not the MOST likely reason for the behavior.

(Correct) The answer to question 110 is C: Rationale:

C is the best answer to create a connection between the group members and provide a sense of security, which will create group cohesion. A is not the best answer to create group cohesion. B focuses more on the individual than on the group process. D may be reasonable at some point, but does not create group cohesion as effectively as C.

(Correct) The answer to question 65 is D: Rationale:

D is correct because any time a client discusses medical symptoms, in this case losing weight and not sleeping, a medical doctor should be referred to for an evaluation. The client also discusses issues regarding domestic violence, thus the social worker should help the client develop a safety plan. A is incorrect because it does not address the medical issues, although it may be appropriate to explore at a later time. B is incorrect because couple's therapy should never occur with a couple where domestic abuse is happening. C is incorrect because a psychiatrist will not be able to address the medical symptoms.

(Correct) The answer to question 18 is D: Rationale:

D is correct because it is the social workers ethical duty to report the potential financial abuse to Adult Protective Services before any other actions are taken. A does not directly address the issue of financial abuse which is the most important problem at this time. B and C would be inappropriate for the social worker to do at this time.

(Correct) The answer to question 54 is D: Rationale:

D is correct because one of the key features of the precontemplation phase of treatment is that the client is not yet ready to admit that they have a problem or need to change their behaviors. A, B, and C are all aspects of the precontemplation phase.

(Correct) The answer to question 56 is D: Rationale:

D is correct because the girl is describing behaviors that may be putting the two girls in danger. Therefore, before any other interventions can be done, the social worker must FIRST call child protective services.

(Correct) The answer to question 11 is D: Rationale:

D is correct because the social worker should FIRST address the values conflict with a supervisor before doing any of the other options. A is inappropriate because it creates an interruption in services by referring the client elsewhere and should only be done if the social worker is unable to manage her own values. B is incorrect because while it may be necessary to involve the client's family at some point in the treatment, providing a family therapy session will not resolve the social worker's value conflict. C is incorrect because it ignores the problem by continuing to meet with the client without addressing the value conflict.

(Correct) The answer to question 125 is D: Rationale:

D is correct because the stem states that the client has only been taking the medication for a week. It generally takes between 2-6 weeks for a medication to start working, therefore the client should be made aware of this before other options are explored at a later time.

(Correct) The answer to question 153 is D: Rationale:

D is correct here because the social worker should FIRST meet with the mother and assess her understanding of the plan and how that relates to the ultimate goal of regaining custody of her children. A would only be done once the social worker has assessed the client's understanding of the plan. At that point creating smaller goals may be addressed if it is deemed necessary. B would not be appropriate at this time because termination of parental rights is a last option, and many steps would need to be done first before that option was explored. C would not be appropriate because confronting the client may make her defensive and would not be an effective intervention.

(Wrong) The answer to question 5 is D: Rationale:

D is the best answer because the child's behavior is well within typical developmental expectations. A is not the best answer because the boy's behavior is not concerning. B is not the best answer because this behavior, in and of itself, does not indicate the child may be transgender. C may be a good intervention and is necessary, but is not as good as D because it does not meet the parents where they are in the moment.

(Correct) The answer to question 57 is D: Rationale:

D is the best answer because the social worker has made a mistake in this situation and this may help repair it. A devalues the client's experience and potentially puts the client in the position of having to reassure the social worker. B is not indicated at this time, nor has it been requested. C is not indicated in this exchange.

(Correct) The answer to question 27 is D: Rationale:

D is the best answer in this case because evaluating a child's response when the caregiver both leaves and returns is drawn from attachment theory. A, B, and C would be indicated by different evaluation techniques.

(Correct) The answer to question 6 is D: Rationale:

D is the best answer. The combination of the information from the referral and the client's behavior in session is most suggestive of a trauma history. A and B are not indicated. The social worker should assess for C during the initial session, but not before D.

(Correct) The answer to question 133 is D: Rationale:

D is the correct answer because it is the only item listed that is NOT part of a mental status exam. A, B, and C are all components of the mental status exam.

(Wrong) The answer to question 9 is D: Rationale:

D is the correct answer because the primary purpose of documentation in this case is to facilitate communication among team members and ensure the clients are receiving proper care.

(Wrong) The answer to question 35 is D: Rationale:

D is the correct answer because this study will collect data at a single point in time from participants of different ages. C would only be the correct answer if the same measure was used at different points in time (point A, 5 years beyond point A, 10 years beyond point A, etc.). A and B do not apply here.

(Wrong) The answer to question 40 is A: Rationale:

If a social worker does not disclose the identity of a client, a release from the client is unnecessary. If a social worker practices in a small rural area where the likelihood of anonymity is low, it would be appropriate for the social worker to obtain a release from their client (A). Making an effort to omit identifying information (B) and conveying the minimal amount of information (C) does not sufficiently protect a client's confidentiality. Altering a client's demographic information (C) could prevent social worker from understanding cultural component that could be contributing to lack of progress.

(Correct) The answer to question 155 is D: Rationale:

If a social worker has reviewed the ethical standards and is still uncertain as to what should be done, consulting with a supervisor would be the most appropriate next step (D). A, B, and C are all incorrect because the social worker needs to consult with someone else before it can be determined what is the most ethical step to take next.

(Wrong) The answer to question 78 is B: Rationale:

In order for a social worker to be culturally competent, he/she must understand how one's own cultural biases influence the manner in which the social worker is able to deal with issues regarding diversity (B).

(Wrong) The answer to question 28 is B: Rationale:

It is important for the social worker to facilitate the client's self-determination. In order to do this, the social worker must first determine what the client needs medically to see if relocating is in her best interest (B). A and D would ignore the client's medical needs. C may be important once the client's medical needs have been assessed.

(Wrong) The answer to question 120 is D: Rationale:

It is important to speak with the client in a non confrontational manner in order to give the client the space to express any problems that he/she may be experiencing with the social worker (D).

(Correct) The answer to question 72 is C: Rationale:

It is unethical to change the client's diagnosis based on the demands of the third party payer, thus options B and D are out. Referring the client to a low fee clinic (A) might be what the social worker ends up doing, but only after discussing payment options with the client (C). Referring him out right away could be seen as abandonment.

(Correct) The answer to question 60 is D: Rationale:

It would be appropriate for the social worker to consult with a colleague to manage countertransference which is noted in the stem and could interfere with treatment (D). The social worker would be imposing his/her own values on the client by expressing a sense of being perplexed (A). It is clear in the stem that the psychiatrist has already discussed the effects of taking xanax during pregnancy (C) and it is not within the social worker's scope of practice to influence medication decisions made by the psychiatrist (B).

(Wrong) The answer to question 167 is D: Rationale:

It would be unethical for the client to accept the furniture at a discounted price and could be viewed as a boundary violation. Therefore, the BEST response for the social worker would be to purchase the furniture at the advertised price (D).

(Correct) The answer to question 74 is D: Rationale:

It's important that the social worker provide the two women with psychoeducation regarding the grief process before anything else is done (D). This allows the family to understand what they may be able to expect as they move through the grief process with their father. A, B, and C, may be appropriate at a later time.

(Wrong) The answer to question 59 is B: Rationale:

It's important to empathize with the parents and acknowledge their frustrations FIRST (B). This helps the parents feel like they are being heard and understood. A, C, and D, may be helpful steps to take only after the social worker has acknowledged how the parents are feeling.

(Correct) The answer to question 83 is D: Rationale:

Making a police report (A) or calling the store (B) would break the client's confidentiality that we are legally required to maintain. social workers are not legally required to break confidentiality and report past acts of violence, thus answer option D is correct. In addition, social workers are not legally required to encourage the clients to turn themselves in, thus C is wrong.

(Correct) The answer to question 103 is A: Rationale:

Many of these answers sound okay. The best answer is A because it includes discussing the boundary violation. Even though it was a text sent to the wrong person, it still included the client into the social worker's personal relationship. B, C, and D, could all be done as well, but the social worker should do A for sure.

(Wrong) The answer to question 29 is A: Rationale:

Most children start to take their first steps between 9 and 12 months of age and can walk well by the time they are 13-14 months old. There is a wide range of what is normal when it comes to first steps. The child is able to be in a standing position while holding a table, which shows the child's first steps are probably not far off. Therefore, A is correct because the mother does not appear to be aware of normal child development and it would be important to first educate the mother.

(Correct) The answer to question 4 is B: Rationale:

Neuroleptics are used to treat (B) psychosis.

(Correct) The answer to question 31 is D: Rationale:

Professional ethics dictates that social workers do not generally see a client who is receiving services from another social worker. Answer (D) allows the social worker to address this conflict directly with client. Telling the boy to no longer see the other social worker (A) interferes with client's self-determination; (B) is not the best answer since we don't know whether or not the client can sign his own release forms; (C) could be correct, but only after speaking with client about conflict.

(Correct) The answer to question 45 is A: Rationale:

Putting things in order, you would want to start with A. It says she is upset - but it isn't clear if it is anger she is feeling or disappointment or sadness. Once you did A, C and D could be done but that would depend on the client's response to A.

(Correct) The answer to question 136 is A: Rationale:

Rather than interrupting the group, the supervisor should allow the group to continue on the topic and explore the concerns with the group leaders afterwards (A). The concerns may be brought back to the group if it is deemed necessary, but since this is not a group problem but more of a group leader problem, it should be addressed with the social workers first.

(Wrong) The answer to question 168 is B: Rationale:

Sexual dysfunction is common during the early stages of recovery. Therefore, it would be important for the social worker to educate the client on the symptoms of withdrawal and what to expect during the early stages of recovery.

(Correct) The answer to question 64 is C: Rationale:

Short-term therapy indicates a limited number of sessions to work on issues with the client. Because the client is being avoidant, the best intervention would be to confront the client regarding this pattern (C). Attempting to explore how client deals with problems outside of session (A) would be ineffective based on client's avoidance. Answers (B) and (D) could be appropriate for long-term, but not short-term therapy.

(Wrong) The answer to question 99 is A: Rationale:

Since the changes in behavior started to occur once the child moved into the family friend's house, it's most important for the social worker to NEXT have a better understanding of what these recent changes look like and how they may be affecting the child (A). B is incorrect because these behavior changes are not necessarily developmentally appropriate for a young child without further assessment. C is incorrect because the social worker does not have enough information to make the assumption that the child is being abused. D is incorrect because the social worker is not aware of what is causing the child's behavioral changes, and therefore moving him to live with another relative may not be fruitful before determining the cause.

(Correct) The answer to question 16 is B: Rationale:

Since the client is in the hospital following a suicide attempt, the social worker is not providing long term therapy. The social worker needs to address the presenting problem, which in this case is the suicide attempt. B is correct because the social worker doesn't want to disregard the information that is given but rather needs to learn how the sexual abuse is contributing to the client's presenting problem. A would be inappropriate to do. C and D would both be unnecessary since it would be irrelevant to know the specifics of the abuse or the client's sexual history.

(Correct) The answer to question 96 is D: Rationale:

Since the client is over the age of 18 it would be unethical for the social worker to disclose any information regarding the client. Therefore, the best response would be to decline providing the information and address the confidential nature of the relationship with the mother (D). While incorporating the mother into treatment may be helpful at a later time, it can only be done if the client with the client's permission.

(Wrong) The answer to question 161 is C: Rationale:

Since the client is underage the social worker must first discuss confidentiality limitations before anything else. The student needs to be aware what the social worker may need to disclose to a parent in the case that the client expresses a desire to hurt himself or others.

(Correct) The answer to question 170 is D: Rationale:

Since the client refuses to go to a shelter, the social worker should next develop a safety plan with the client in case her partner continues to be physically abusive towards her. A and B may be explored at a later point. C should not be done because couple's therapy should never be provided to a couple where domestic violence is occurring as it can worsen the situation.

(Wrong) The answer to question 156 is C: Rationale:

Since the social worker has already completed research on the best approach for the group, the NEXT step would be to determine a way to measure how effective the group actually is in teaching social skills to kids with Autism Spectrum Disorder (C). A, B, and D may all be steps taken later, but not until the social worker has established a plan to measure effectiveness.

(Correct) The answer to question 166 is B: Rationale:

Since the social worker is new to the field and not as knowledgeable about resources, the social worker should consult with a colleague who is more educated on the appropriate resources. The other options may be appropriate steps for the social worker to take after a knowledgeable colleague has been consulted.

(Correct) The answer to question 21 is A: Rationale:

Since the stem states that the client has limited family connections, it is necessary that the client connect with community agencies as he transitions out of the hospital (A). B, C, and D may be important to do at a later time but only after a connection has been made with other community agencies.

(Correct) The answer to question 160 is D: Rationale:

Since these behavioral and academic problems appear to be a new problem for the student, it's important for the social worker to explore what changed for the student or prompted this. Therefore, D must be done FIRST to determine what steps should be taken next.

(Correct) The answer to question 39 is B: Rationale:

The BEST response for the social worker is to address the social worker's difficulties with a supervisor and how they may be affecting the work with the client (B). C would be inappropriate. D would interrupt the therapeutic relationship, which may not be necessary without first consulting with a supervisor. A would be incorrect because there is nothing in the stem stating that the colleague has the appropriate feedback or experience in this type of situation.

(Correct) The answer to question 84 is C: Rationale:

The Code of Ethics states that a social worker should refer clients to other professionals when the other professionals knowledge or expertise is needed to fully serve the client. The social worker in this case does not have the appropriate knowledge to best serve this client and should therefore refer the client to a therapist who is knowledgeable in that area (C).

(Correct) The answer to question 90 is C: Rationale:

The Code of Ethics states that social workers should seek consultation with a supervisor when faced with personal problems that are affecting their work in order to protect their clients (C). A, B, and D may be done after the social worker has consulted with a supervisor if it is deemed appropriate.

(Correct) The answer to question 46 is D: Rationale:

The MOST appropriate response here is D because it encourages the client to explain how she views the problem. The social worker may think the client understands, however, the client may be viewing from a different angle and therefore A would be unnecessary because it does not address the client's behaviors. B and C don't address the client's reaction to the social worker although they may be important to do after the client's view of the problem has been explored.

(Wrong) The answer to question 113 is A: Rationale:

The best answer is A. Similar to couples therapy, you want to direct the person to bring the issue up in the group session. As the leader you would help facilitate that discussion but you would not want to be the one who brings it up as it would indicate that the leader has had outside discussions with a group member. You could also do C as part of the discussion, but you would want to start with A.

(Correct) The answer to question 150 is C: Rationale:

The best answer is C. It is important to explore the significance of her question because it will clarify the cultural issues at play for her. All the other answers bypass exploring the significance.

(Wrong) The answer to question 63 is C: Rationale:

The best answer is C. The information shared by the boy points to sexual abuse, which requires the social worker to make a child abuse report, answer C. Answers A, B, and D are incorrect because they do not address the legal mandate to report abuse.

(Correct) The answer to question 89 is C: Rationale:

The client has presented a legal issue to the social worker, which is outside the social worker's scope of practice. Therefore, the social worker would best serve the client by encouraging her to seek legal advice by someone qualified to provide those services (C). Encouraging the client to request disability insurance (A) or exploring alternative employment options (B) do not address the potential legal issues that arise in this case. Confronting the manager could be detrimental to the client if she is to pursue legal action (D).

(Correct) The answer to question 34 is A: Rationale:

The correct answer is A. This is a recall question. Out of the answer options available, Trauma Focused Cognitive Behavioral Therapy (TF-CBT) would be the most appropriate treatment for a teenager who has a trauma history. TF-CBT is an evidence-based treatment for children and adolescents used to treat the effects of trauma.

(Wrong) The answer to question 114 is A: Rationale:

The correct answer is A. While a client with Schizophrenia could also be diagnosed with a depressive or anxiety disorder, the MOST commonly diagnosed co-occurring disorder with Schizophrenia is Substance Use Disorder. Many individuals with Schizophrenia use substances in an attempt to self-medicate.

(Correct) The answer to question 152 is C: Rationale:

The correct answer is C because prior to referring the client to NA (B), or for a psychiatric evaluation (A), and even prior to continuing with assessment (D) the social worker should first educate the client on the nature of his / her symptoms. Understanding that repeated use of cocaine depletes the brain of those neurotransmitters necessary to sustain normal mood, resulting in depression will help the client grasp and deal with the reality of the situation.

(Wrong) The answer to question 43 is D: Rationale:

The correct answer is D. Even if contacted by the police, the social worker is not required to disclose information about a deceased client (B). The NASW Code of Ethics requires that we use the same standards of confidentiality for deceased clients as we do for living clients. It allows us to release information only with the consent of a person legally authorized to consent on behalf of the client. There is no reason to consult with a supervisor since the law is clear and there is no indication the social worker is uncertain how to proceed (A). (C) is incorrect because it is too absolute. There are circumstances in which we can share this information, including when asked to do so by the client's legal representative.

(Wrong) The answer to question 116 is D: Rationale:

The first task of the social worker is to identify and prioritize the client's needs, and then link them with services to meet those needs (D). Providing information regarding financial services (C) could be part of such intervention, but there are likely to be other services that client should be linked to. There is not enough information to indicate neglect (B). When the immediate needs have been met, the social worker can begin to help the client develop skills necessary to deal with stressors (A).

(Correct) The answer to question 108 is B: Rationale:

The immediate concern for the social worker should be the client's potential for self-harm or harm to others and working toward an emergency psychiatric commitment (B) would address this priority. Encouraging the father to file a police report (A) or assisting the parents in filing a restraining order (C) does not address the current needs of the client. Due to the psychotic features and agitation, attempting to convince the client to take his medication (D) would be ineffective and does not address the client's immediate danger to self and others.

(Correct) The answer to question 71 is C: Rationale:

The physician is completely disregarding the client's right to self-determination which is why C is correct. The social worker needs to state that the client has a right to be a part of her discharge plans.

(Correct) The answer to question 159 is B: Rationale:

The previously substantiated report of abuse is the most pertinent information because there is a greater chance of recurrence in child abuse when the family has a history of abuse (B).

(Wrong) The answer to question 127 is B: Rationale:

The question stem is describing a process recording. Written records of client interactions for educational and learning purposes that do not go into the client's official record are described as process recordings.

(Wrong) The answer to question 107 is B: Rationale:

The reason B is the answer is that there is no indication of the cash value of the card so the best response would be to decline the card. The other issue is that unlike other types of gifts that may have cultural meaning, a gift card is essentially cash. If a client tried to pay you a "bonus" for doing your job, it would be unethical to accept that.

(Correct) The answer to question 13 is A: Rationale:

The social worker always wants to encourage the client's sense of self-determination; therefore, it's important for the social worker to allow the client to decide what is best for him (A). B and D disregard the client's statement that he does not want to take medication. C may be important to explore after A is done.

(Correct) The answer to question 17 is D: Rationale:

The social worker has implemented a protocol that ensures only trained mental health professionals will do formal risk evaluations for suicidal students, therefore making D incorrect. What is being described in the question stem is not empowering all staff to evaluate students for suicide risk.

(Correct) The answer to question 130 is C: Rationale:

The social worker needs to call 911 before any further actions are taken (C) so that the client can be assessed by a medical professional. A is incorrect because the client already took the pills, therefore a consent is not needed. Finishing the assessment would be inappropriate (B) because the client needs to be assessed immediately. D is incorrect because it would be inappropriate for the social worker to bring the client to a hospital herself.

(Correct) The answer to question 88 is C: Rationale:

The social worker needs to focus on the presenting problem (anxiety) and respect the client's beliefs even if she does not agree with them (C). A is incorrect because it is the social worker's job to maintain her own value system and therefore providing the client with outside referrals would create an unnecessary interruption of services. B and D would be inappropriate to do.

(Correct) The answer to question 36 is B: Rationale:

The social worker needs to understand what affects the client's behavior in order to identify issues to be addressed in therapy and gathering a social history would provide the social worker with this information (B). Answers (C) is only one component of a client's social history. Facilitating a referral (A) and understanding coping mechanisms (D) are not included in a client's social history, but rather are included in the assessment and referral phase.

(Correct) The answer to question 105 is C: Rationale:

The social worker should FIRST discuss the cycle of violence, and the dynamics of domestic abuse (C) with the client before doing any further interventions. A is incorrect because marital therapy should never be provided to a couple where domestic abuse is occurring. B is incorrect because the client is describing domestic violence issues, not communication issues. D may be appropriate after the social worker has explored the dynamics of domestic abuse with the client.

(Wrong) The answer to question 37 is D: Rationale:

The social worker should FIRST meet with the parents and child for an assessment and to discuss the problem from their perspective (D). There is no discussion in the stem of a need for the child to see the school nurse (A). B may be done at a later date if it is determined that child abuse may be occurring. C may also be done at a later date but only once the social worker has met with the parents and child.

(Wrong) The answer to question 137 is D: Rationale:

The social worker should explore with the client whether or not he/she understands the payment for service agreement before any actions are further actions are taken (D). Once this has been explored the social worker can then determine the most appropriate next step.

(Wrong) The answer to question 158 is A: Rationale:

The social worker should first explain confidentiality to both the son and the parents (A) in order to clarify the role of the social worker and help the son understand how the information he shares in therapy is protected. Answers (B) and (C) only address either the parents or the son, but not both. The social worker should validate the son's concerns about being understood (D), but only after explaining confidentiality to the entire family.

(Correct) The answer to question 128 is B: Rationale:

The social worker should report the client since he states an intent to assault someone else (B). A is incorrect because the client has stated an intent which means that future harm may be committed. In this case, confidentiality can be broken. C and D would not be appropriate at this time since the client has stated an intention to harm another person, therefore reporting the intent takes priority.

(Correct) The answer to question 25 is D: Rationale:

The social worker should send (D), portions of the record, when asked for a summary. You would not send the whole record (A). And only medications or progress notes would be inadequate.

(Correct) The answer to question 118 is B: Rationale:

The social worker should start where the client is by addressing how the client is thinking and feeling about this major life change. A and C may be helpful at some point, but would not occur before the social worker addresses the client's thoughts and feelings. D does not do anything to respond to the client's expressed feelings.

(Wrong) The answer to question 104 is B: Rationale:

The son's concerns may be appropriate, but it is best to FIRST explore his concerns and see if they can be remedied in any way (B). A would be appropriate at a later time. C is unnecessary because nothing in the stem shows that a consultation with a supervisor is necessary. D is incorrect because the patient's desire to live with his son has not been explored yet and the patient does not express a desire to do inpatient hospice care.

(Correct) The answer to question 22 is C: Rationale:

The stem makes it clear that the mother has made no progress in improving her behavior despite extensive casework services. With this in mind, it is important for the foster care worker to prepare the children for the next step, which would be having parental rights terminated due to the lack of parental progress (C). A and D are incorrect because the stem states that extensive efforts have already been made to change the mother's behaviors. B may be done at a later date if deemed necessary but would not be the NEXT step.

(Correct) The answer to question 52 is D: Rationale:

There are enough signs to reasonably suspect child abuse. Thus, the social worker working from any theoretical perspective should first and foremost make a child abuse report (D).

(Wrong) The answer to question 8 is B: Rationale:

This is a factual recall question. What is being described in the question stem is a SOAP (Subjective, Objective, Assessment, Plan) note (B). A SOAP note is a type of progress note (A). A progress note is a more general term for notes that are part of the client's written record and offer details about the client's presenting issue/symptoms, diagnosis, assessment, treatment plan, progress toward goals. (B) and (D) are two types of progress notes. Process notes (C) is not a type of progress note. These are personal notes taken by a social worker that are not part of the client's formal record, but are taken to help the social worker best help their client. They include things like hypotheses and notes/questions for reflection and consultation. A DAP note (D) is a type of progress note that stands for data, assessment, and planning

(Correct) The answer to question 53 is B: Rationale:

This is a recall question. B is the best answer because the subject matter and the client's affect do not line up or make sense. A would be an appropriate answer if the subject matter and the client's affect/response were in line with one another. C may be true, but B is a better and more precise description of what is happening. D is not indicated.

(Correct) The answer to question 55 is A: Rationale:

This is a recall question; A is the best answer because it is common to encounter the emotional process of shame when working with Asian clients; this can sometimes prevent or delay clients from seeking help, or may discourage clients from engaging in discussions around emotional content. B, C, and D are not commonly associated with working with Asian clients.

(Correct) The answer to question 139 is A: Rationale:

This is a recall question; A is the best answer because the stem describes 2 components of DBT along with the qualifier that the approach is "structured." Mindfulness activities are an essential component of DBT. B, C, and D would all be described differently.

(Wrong) The answer to question 44 is A: Rationale:

This question is asking what the social worker should do first. Providing the client with honest information regarding credentials (A) would help build rapport with client. After sharing information, the social worker could ask the client how he/she feels about seeking help (C) as well as explain history of working with similarly diagnosed clients (D). Confronting clients (B) is not an intervention used in the early stages of therapy and could be detrimental to the client-social worker relationship.

(Wrong) The answer to question 42 is A: Rationale:

This question is asking what the social worker should do first. The social worker should first take a history of the client's problems (A). The social worker should obtain a signed release to speak with the client's doctor (C) and in the future could refer for psychiatric evaluation (B), but only after taking a history. Referring a client to an exercise program (D) is not clearly indicated in the stem as an appropriate referral and the social worker should consult with the doctor to discuss this type of referral due to client's health issues.

(Correct) The answer to question 109 is D: Rationale:

This question is asking what the social worker should do first. The social worker should validate and acknowledge the client's feelings (D) prior to taking any additional steps. The social worker could choose to use any of the other interventions listed in answers A through C, but only after acknowledging the client's feelings.

(Correct) The answer to question 143 is D: Rationale:

This question is asking what the social worker should do next. The first goal of the social worker should be to clarify boundaries (D) then do (C), since there is evidence the client is unaware of clinical boundaries. The social worker could then do (B). There is no indication that the therapist must refer out, but the boundary issues should be reviewed.

(Correct) The answer to question 48 is C: Rationale:

This would be reportable if the mother was still alive, but since she isn't the best answer is C. You could also do B and D at some point but you'd want to start where the client is.

(Correct) The answer to question 32 is C: Rationale:

This would trigger a mandated report for neglect so C is the best answer. There is enough for reasonable suspicion so D is not indicated. A and B could be done at some point but a report has been triggered so that needs to be done first.

(Correct) The answer to question 92 is B: Rationale:

When a client asks a personal question like this, it's important to always explore FIRST why the information is important to the client (B).

(Correct) The answer to question 20 is C: Rationale:

When a client presents with recent physical or emotional changes, it is always important for the client to FIRST see a doctor for a physical exam (C). A, B, and D may be appropriate interventions after a medical exam has ruled out any biological causes for the change.

(Correct) The answer to question 68 is C: Rationale:

When an ethical violation occurs, it is important for the social worker to FIRST speak with the colleague in order to discuss the boundary violation before taking further action. (A), (B), and (D) may be done at a later date once the social worker has spoken with the colleague.

(Correct) The answer to question 98 is B: Rationale:

When creating objectives, they need to be measurable and achievable. B is correct because it allows the client to work towards a goal that is not all black or white. Arguments are a normal part of adolescence and therefore no more than two arguments per week is obtainable for the client to work towards. The goal is also measurable so the client and social worker can see how well he is working towards that goal each week. A is too broad of a goal and not specific enough to be measured. C is incorrect because there is no way to measure what 75 percent of the time would be. D is incorrect because it is unreasonable goal and only sets the client up for failure.

(Correct) The answer to question 134 is C: Rationale:

When terminating with a client it's important to make sure that the client is able to use what has been in treatment and apply that to their life (C). Until that question can be answered confidently, the client should continue to see the therapist.

(Correct) The answer to question 146 is A: Rationale:

When using electronic communication the client should always be aware of the risks involved and the limitations in comparison to speaking in person. While B and C may be explored with the client, the social worker should FIRST respond to the client's interest in using email to communicate with the social worker and what risks this may entail. D would not be beneficial to the client.

(Correct) The answer to question 112 is B: Rationale:

When working with clients who are paranoid it's important to establish a trusting relationship and good rapport in order to engage the client in treatment (B). A may be important to explore at a later time, but the stem states that the client is currently refusing substance abuse treatment so it would be futile to explore that option until a therapeutic alliance has developed. C would be important to do after a trusting relationship has been established. D is incorrect because a client who is paranoid may view this as being confrontational and it would not be an effective intervention.

(Correct) The answer to question 122 is D: Rationale:

When writing case notes it is important for the social worker to state how the client is working towards his/her service goals (D) so that the court is aware of how the client is progressing and meeting the previously agreed on objectives.

(Wrong) The answer to question 95 is D: Rationale:

While all of these answer options could take place at some point, the FIRST thing we should do is (D). The best place to start when concerned about discrimination taking place is to document the instances of discrimination that are taking place. This provides the social worker with reference materials they can refer to if and when taking further steps.

(Correct) The answer to question 135 is C: Rationale:

While spanking is not always considered child abuse, if a mark, bruise, scar, or wound is left on the child, it MUST be reported to the appropriate agency for further investigation (C).


संबंधित स्टडी सेट्स

Health and Life (chapters 8 & 9)

View Set

Ch 7 CMA Exam Review - Nutrition

View Set

Urinalysis & Other Body Fluids (Review)

View Set

6.EE.2c Evaluate expressions given a value for the variable

View Set

Principles of Computer Programming Vocabulary

View Set